Buy BOOKS at Discounted Price

The Plane

Class 12th Mathematics RS Aggarwal Solution
Exercise 28a
  1. Find the equation of the plane passing through each group of points:(i) A(2, 2,…
  2. Show that the four points A(3, 2, -5),B(-1, 4, -3), C(-3, 8, -5) and D(-3, 2,…
  3. Show that the four points A(0, -1, 0),B(2, 1, -1), C(1, 1, 1) and D(3, 3, 0)…
  4. Write the equation of the plane whose intercepts on the coordinate axes are 2,…
  5. Reduce the equation of the plane 4x – 3y + 2z = 12 to the intercept form, and…
  6. Find the equation of the plane which passes through the point (2, -3,7) and…
  7. A plane meets the coordinate axes at A, B and C respectively such that the…
  8. Find the Cartesian and vector equations of a plane passing through the point…
  9. If O is the origin and P(1, 2, -3) be a given point, then find the equation of…
Exercise 28j
  1. Find the direction ratios of the normal to the plane x + 2y - 3z = 5.…
  2. Find the direction cosines of the normal to the plane 2x + 3y - z = 4.…
  3. Find the direction cosines of the normal to the plane y = 3.
  4. Find the direction cosines of the normal to the plane 3x + 4 = 0.…
  5. Write the equation of the plane parallel to XY-plane and passing through the…
  6. Write the equation of the plane parallel to YZ-plane and passing through the…
  7. Write the general equation of a plane parallel to the x-axis.
  8. Write the intercept cut off by the plane 2x + y - z = 5 on the x-axis.…
  9. Write the intercepts made by the plane4x - 3y + 2z = 12 on the coordinate axes.…
  10. Reduce the equation 2x – 3y + 5z + 4 = 0 to intercept form and find the…
  11. Find the equation of a plane passing through the points A(a, 0, 0), B(0, b, 0)…
  12. Write the value of k for which the planes 2x – 5y + kz = 4 and x + 2y – z = 6…
  13. Find the angle between the planes 2x + y – 2z = 5 and 3x – 6y – 2z = 7.…
  14. Find the angle between the planes and vector {r} c. ( {i} + hat{k} ) = 3…
  15. Find the angle between the planes vector {r} c. ( 3 {i}-4 hat { jmath } + 5…
  16. Find the angle between the line {x+1}/{2} = frac {y}/{3} = frac {z-3}/{6}…
  17. Find the angle between the line and the plane vector {r} c. ( 2i - {j} +…
  18. Find the value of λ such that the line {x-2}/{6} = frac {y-1}/{2} = frac…
  19. Write the equation of the plane passing through the point (a, b, c) and…
  20. Find the length of perpendicular drawn from the origin to the plane 2x – 3y +…
  21. Find the direction cosines of the perpendicular from the origin to the plane…
  22. Show that the line vector {r} = ( 4i^{0} - 7 {k} ) / i ( 4i^{2} - 2j+3k…
  23. Find the length of perpendicular from the origin to the plane
  24. Find the value of λ for which the line is parallel to the plane
  25. Write the angle between the line and the plane x + y + 4 = 0.
  26. Write the equation of a plane passing through the point (2, -1, 1) and…
Objective Questions
  1. The direction cosines of the perpendicular from the origin to the plane vector {r} c.…
  2. The direction cosines of the normal to the plane 5y + 4 = 0 are Mark against the…
  3. The length of perpendicular from the origin to the plane vector {r} c. ( 3 {i}-4…
  4. The equation of a plane passing through the point A(2, -3, 7) and making equal…
  5. A plane cuts off intercepts 3, -4, 6 on the coordinate axes. The length of…
  6. If the line {x+1}/{3} = frac {y-2}/{4} = frac {z+6}/{5} is parallel to the plane…
  7. If O is the origin and P(1, 2, -3) is a given point, then the equation of the plane…
  8. If the line {x-4}/{1} = frac {y-2}/{1} = frac {z-k}/{2} lies in the plane 2x – 4y…
  9. The plane 2x + 3y + 4z =12 meets the coordinate axes in A, B and C. The centroid of…
  10. If a plane meets the coordinate axes in A, B and C such that the centroid of ∆ABC is…
  11. The equation of a plane through the point A(1, 0, -1) and perpendicular to the line…
  12. The line meets the plane 2x + 3y – z = 14 in the point Mark against the correct…
  13. The equation of the plane passing through the points A(2, 2, 1) and B(9, 3, 6) and…
  14. The equation of the plane passing through the intersection of the planes 3x - y + 2z –…
  15. The equation of the plane passing through the points A(0, -1, 0), B(2, 1, -1) and C(1,…
  16. If the plane 2x – y + z = 0 is parallel to the line {2x-1}/{2} = frac {2-y}/{2} =…
  17. The angle between the line {x+1}/{1} = frac {y}/{2} = frac {z-1}/{1} and a…
  18. The point of intersection of the line {x-1}/{3} = frac {y+2}/{4} = frac {z-3}/{-2}…
  19. The equation of a plane passing through the points A(a, 0, 0), B(0, b, 0) and C(0, 0,…
  20. If θ is the angle between the planes 2x – y + 2z = 3 and 6x – 2y + 3z = 5, then cos θ…
  21. The angle between the planes 2x – y + z = 6 and x + y + 2z = 7, is Mark against the…
  22. The angle between the planes vector {r} c. ( 3 {i}-6 hat{j}+2 hat{k} ) = 4 and…
  23. The equation of the plane through the points A(2, 3, 1) and B(4, -5, 3), parallel to…
  24. A variable plane moves so that the sum of the reciprocals of its intercepts on the…
  25. The equation of a plane which is perpendicular to ( 2 {i} - hat{3} hat{j} + hat{k}…
  26. The equation of the plane passing through the point A(2, 3,4) and parallel to the…
  27. The foot of the perpendicular from the point A(7, 14, 5) to the plane 2x + 4y – z = 2…
  28. The equation of the plane which makes with the coordinate axes, a triangle with…
  29. The intercepts made by the plane vector {r} c. ( 2 {i}-3 hat{j}+4 hat{k} ) = 12…
  30. The angle between the line {x-2}/{1} = frac {y+3}/{-2} = frac {z+4}/{-3} and the…
  31. The angle between the line vector {r} c. ( {i} + hat{j}-3 hat{k} ) + lambda ( 2…
  32. The distance of the point ( {i}+2 hat{j}+5 hat{k} ) from the plane vector {r} c.…
  33. The distance between the parallel planes 2x – 3y + 6z = 5 and 6x – 9y + 18z + 20 = 0,…
  34. The distance between the planes x + 2y – 2z + 1 = 0 and 2x + 4y – 4z – 4z + 5 = 0, is…
  35. The image of the point P(1, 3, 4) in the plane 2x – y + z + 3 = 0, is Mark against the…
Exercise 28b
  1. Find the vector and Cartesian equations of a plane which is at a distance of 5…
  2. Find the vector and Cartesian equations of a plane which is at a distance of 7…
  3. Find the vector and Cartesian equations of a plane which is at a distance of…
  4. Find the vector and Cartesian equations of a plane which is at a distance of 6…
  5. Find the vector, and Cartesian equations of a plane which passes through the…
  6. Find the length of the perpendicular from the origin to the plane f c.…
  7. Find the Cartesian equation of the plane whose vector equation is vector {r}…
  8. Find the vector equation of a plane whose Cartesian equation is 5x - 7y + 2z +…
  9. Find a unit vector normal to the planex – 2y + 2z = 6.
  10. Find the direction cosines of the normal to the plane 3x – 6y + 2z = 7.…
  11. For each of the following planes, find the direction cosines of the normal to…
  12. Find the vector and Cartesian equations of the plane passing through the point…
  13. Find the coordinates of the foot of the perpendicular drawn from the origin to…
  14. Find the length and the foot of perpendicular drawn from the point (2, 3, 7)…
  15. Find the length and the foot of the perpendicular drawn from the point (1, 1,…
  16. From the point P(1, 2, 4), a perpendicular is drawn on the plane 2x + y - 2z +…
  17. Find the coordinates of the foot of the perpendicular and the perpendicular…
  18. Find the coordinates of the image of the point P(1, 3, 4) in the plane 2x - y…
  19. Find the point where the line {x-1}/{2} = frac {y-2}/{-3} = frac {z+3}/{4}…
  20. Find the coordinates of the point where the line through (3, -4, -5) and (2,…
  21. Find the distance of the point (2, 3, 4) from the plane 3x + 2y + 2z +5 =0,…
  22. Find the distance of the point (0, -3, 2) from the plane x + 2y -z = 1,…
  23. Find the equation of the line passing through the point P(4, 6, 2) and the…
  24. Show that the distance of the point of intersection of the line {x-2}/{3}…
  25. Find the distance of the point A(-1, -5, -10) from the point of intersection…
  26. Prove that the normals to the planes 4x + 11y + 2z + 3 = 0 and 3x - 2y + 5z =…
  27. Show that the line vector {r} = ( 2 {i}-2 hat{j}+3 hat{k} ) + lambda ( i…
  28. Find the equation of a plane which is at a distance of 3√3 units from the…
  29. A vector bar {n} of magnitude 8 units is inclined to the x-axis at 45o,…
  30. Find the vector equation of a line passing through the point ( 2 {i}-3…
Exercise 28c
  1. Find the distance of the point ( 2 {i} - hat{j}-4 hat{k} ) from the plane…
  2. Find the distance of the point ( {i}+2 hat{j}+5 hat{k} ) from the plane…
  3. Find the distance of the point (3, 4, 5) from the plane vector {r} c. ( 2…
  4. Find the distance of the point (1, 1, 2) from the plane vector {r} c. ( 2…
  5. Find the distance of the point (2, 1, 0) from the plane 2x + y + 2z + 5 = 0.…
  6. Find the distance of the point (2, 1, - 1) from the plane x – 2y + 4z = 9.…
  7. Show that the point (1, 2, 1) is equidistant from the planes vector {r} c. (…
  8. Show that the points ( - 3, 0, 1) and (1, 1, 1) are equidistant from the plane…
  9. Find the distance between the parallel planes 2x + 3y + 4 = 4 and 4x + 6y + 8z…
  10. Find the distance between the parallel planes x + 2y - 2z + 4 = 0 and x + 2y –…
  11. Find the equation of the planes parallel to the plane x – 2y + 2z – 3 = 0,…
  12. Find the equation of the plane parallel to the plane 2x – 3y + 5z + 7 = 0 and…
  13. Find the equation of the plane mid - parallel to the planes 2x – 3y + 6z + 21 = 0 and 2x –…
Exercise 28d
  1. Show that the planes 2x – y + 6z = 5 and 5x – 2.5y + 15z = 12 are parallel.…
  2. Find the vector equation of the plane through the point ( 3 {i}+4 hat{j} -…
  3. Find the vector equation of the plane passing through the point (a, b, b) and…
  4. Find the vector equation of the plane passing through the point (1, 1, 1) and…
  5. Find the equation of the plane passing through the point (1, 4, - 2) and…
  6. Find the equations of the plane passing through the origin and parallel to the…
  7. Find the equations of the plane passing through the point ( - 1, 0, 7) and…
  8. Find the equations of planes parallel to the plane x – 2y + 2z = 3 which are at…
  9. Find the distance between the planes x + 2y + 3z + 7 = 0 and 2x + 4y + 6z + 7 =…
Exercise 28e
  1. Find the equation of the plane through the line of intersection of the planes x…
  2. Find the equation of the plane through the line of intersection of the planes x…
  3. Find the equation of the plane passing through the intersection of the planes…
  4. Find the equation of the plane passing through the line of intersection of the…
  5. Find the equation of the plane passing through the intersection of the planes x…
  6. Find the equation of the plane passing through the line of intersection of the…
  7. Find the equation of the plane through the intersection of the planes 3x – 4y +…
  8. Find the vector equation of the plane through the intersection of the planes…
  9. Find the vector equation of the plane through the point (1, 1, 1), and passing…
  10. Find the vector equation of the plane passing through the intersection of the…
  11. Find the equation of the plane through the line of intersection of the planes…
  12. Find the Cartesian and vector equations of the planes through the line of…
Exercise 28f
  1. Find the acute angle between the following planes :(i) and vector {f} c. ( 2…
  2. Show that the following planes are at right angles:(i) vector {r} c. ( 4…
  3. Find the value of λ for which the given planes are perpendicular to each…
  4. Find the acute angle between the following planes:(i) 2X – y + z = 5 and x + y…
  5. Show that each of the following pairs of planes are at right angles:(i) 3x + 4y…
  6. Prove that the plane 2x + 2y + 4z = 9 is perpendicular to each of the planes x…
  7. Show that the planes 2x – 2y + 4z + 5 = 0 and 3x – 3y + 6z - 1 = 0 are…
  8. Find the value of λ for which the planes x – 4y + λz + 3 = 0 and 2x + 2y + 3z =…
  9. Write the equation of the plane passing through the origin and parallel to the…
  10. Find the equation of the plane passing through the point (a, b, c) and…
  11. Find the equation of the plane passing through the point (1, -2, 7) and…
  12. Find the equation of the plane passing through the point A(-1, -1, 2) and…
  13. Find the equation of the plane passing through the origin and perpendicular to…
  14. Find the equation of the plane that contains the point A(1, -1, 2) and is…
  15. Find the equation of the plane passing through the points A(1, 1, 2) and B(2,…
  16. Find the equation of the plane passing through the points A(-1, 1, 1) and B(1,…
  17. Find the equation of the plane through the points A( 3, 4, 2) and B(7, 0, 6)…
  18. Find the equation of the plane through the points A(2, 1, -1) and B(-1, 3, 4)…
Exercise 28g
  1. Find the angle between the line and the plane vector {r} c. ( 2 {i} -…
  2. Find the angle between the line vector {r} = ( 2 {i} - hat{j}+3 hat{k} ) +…
  3. Find the angle between the line vector {r} = ( 3 {i} + hat{k} ) + lambda (…
  4. Find the angle between the line {x-2}/{3} = frac {y+1}/{-1} = frac…
  5. Find the angle between the line {x+1}/{2} = frac {y}/{3} = frac {z-3}/{6}…
  6. Find the angle between the line joining the points A(3, - 4, - 2) and B(12, 2,…
  7. If the plane 2x – 3y – 6z = 13 makes an angle sin - 1 (λ) with the x - axis,…
  8. Show that the line vector {r} = ( 2 {i}+5 hat{j}+7 hat{k} ) + lambda (…
  9. Find the value of m for which the line vector {r} = ( {i}+2 hat{k} ) + lambda…
  10. Find the vector equation of a line passing through the origin and…
  11. Find the vector equation of the line passing through the point with position…
  12. Show that the equation ax + by + d = 0 represents a plane parallel to the z -…
  13. Find the equation of the plane passing through the points (1, 2, 3) and (0, -…
  14. Find the equation of a plane passing through the point (2, - 1, 5),…
  15. Find the equation of the plane passing through the intersection of the…
Exercise 28h
  1. Find the vector and Cartesian equations of the plane passing through the origin…
  2. Find the vector equation of a plane passing through the point (1, 2, 3) and…
  3. Find the vector and Cartesian equations of the plane passing through the…
  4. Find the Cartesian and vector equations of a plane passing through the point…
  5. Find the vector equation of the plane passing through the point ( 3 {i}+4…
Exercise 28i
  1. Show that the lines vector {r} = ( 2 {j}-3 hat{k} ) + lambda ( {i}+2…
  2. Find the vector and Cartesian forms of the equations of the plane containing…
  3. Find the vector and Cartesian equations of a plane containing the two lines…
  4. Prove that the lines {x}/{1} = frac {y-2}/{2} = frac {z+3}/{3} and…
  5. Prove that the lines {x-2}/{1} = frac {y-4}/{4} = frac {z-6}/{7} and…
  6. Show that the lines {5-x}/{-4} = frac {y-7}/{4} = frac {z+3}/{-5} and…
  7. Show that the lines {x+1}/{-3} = frac {y-3}/{2} = frac {z+2}/{1} and…
  8. Show that the lines {x-1}/{2} = frac {y-3}/{-1} = frac {z}/{-1} and…
  9. Find the equation of the plane which contains two parallel lines given by…

Exercise 28a
Question 1.

Find the equation of the plane passing through each group of points:

(i) A(2, 2, -1), B(3, 4, 2) and C(7, 0, 6)

(ii) A(0, -1, -1), B(4, 5, 1) and C(3, 9, 4)

(iii) A(-2, 6, -6), B(-3, 10, 9) and



Answer:

(i) A(2, 2, -1), B(3, 4, 2) and C(7, 0, 6)


Given Points :


A = (2, 2, -1)


B = (3, 4, 2)


C = (7, 0, 6)


To Find : Equation of plane passing through points A, B & C


Formulae :


1) Position vectors :


If A is a point having co-ordinates (a1, a2, a3), then its position vector is given by,



2) Vector :


If A and B be two points with position vectors respectively, where




then,




3) Cross Product :


If are two vectors




then,



4) Dot Product :


If are two vectors




then,



5) Equation of Plane :


If A = (a1, a2, a3), B = (b1, b2, b3), C = (c1, c2, c3) are three non-collinear points,


Then, the vector equation of the plane passing through these points is



Where,



For given points,


A = (2, 2, -1)


B = (3, 4, 2)


C = (7, 0, 6)


Position vectors are given by,





Now, vectors are








Therefore,






Now,



= 40 + 16 + 12


= 68


………eq(1)


And



= 20x + 8y – 12z


………eq(2)


Vector equation of the plane passing through points A, B & C is



From eq(1) and eq(2)


20x + 8y – 12z = 68


This is 5x + 2y – 3z = 17 vector equation of required plane.


(ii) Given Points :


A = (0, -1, -1)


B = (4, 5, 1)


C = (3, 9, 4)


To Find : Equation of plane passing through points A, B & C


Formulae :


1) Position vectors :


If A is a point having co-ordinates (a1, a2, a3), then its position vector is given by,



2) Vector :


If A and B be two points with position vectors respectively, where




then,




3) Cross Product :


If are two vectors




then,



4) Dot Product :


If are two vectors




then,



5) Equation of Plane :


If A = (a1, a2, a3), B = (b1, b2, b3), C = (c1, c2, c3) are three non-collinear points,


Then, vector equation of the plane passing through these points is



Where,



For given points,


A = (0, -1, -1)


B = (4, 5, 1)


C = (3, 9, 4)


Position vectors are given by,





Now, vectors are








Therefore,






Now,



= 0 + 14 – 22


= - 8


………eq(1)


And



= 10x - 14y + 22z


………eq(2)


Vector equation of plane passing through points A, B & C is



From eq(1) and eq(2)


10x - 14y + 22z = - 8


This is 5x - 7y + 11z = - 4 vector equation of required plane


(iii) Given Points :


A = (-2, 6, -6)


B = (-3, 10, 9)


C = (-5, 0, -6)


To Find : Equation of plane passing through points A, B & C


Formulae :


1) Position vectors :


If A is a point having co-ordinates (a1, a2, a3), then its position vector is given by,



2) Vector :


If A and B be two points with position vectors respectively, where




then,




3) Cross Product :


If are two vectors




then,



4) Dot Product :


If are two vectors




then,



5) Equation of Plane :


If A = (a1, a2, a3), B = (b1, b2, b3), C = (c1, c2, c3) are three non-collinear points,


Then, vector equation of the plane passing through these points is



Where,



For given points,


A = (-2, 6, -6)


B = (-3, 10, 9)


C = (-5, 0, -6)


Position vectors are given by,





Now, vectors are








Therefore,






Now,



= - 180 - 270 – 108


= - 558


………eq(1)


And



= 90x - 45y + 18z


………eq(2)


Vector equation of plane passing through points A, B & C is



From eq(1) and eq(2)


90x - 45y + 18z = - 558


This is 10x - 5y + 2z = - 62 vector equation of required plane



Question 2.

Show that the four points A(3, 2, -5),
B(-1, 4, -3), C(-3, 8, -5) and D(-3, 2, 1) are coplanar. Find the equation of the plane containing them.


Answer:

Given Points :


A = (3, 2, -5)


B = (-1, 4, -3)


C = (-3, 8, -5)


D = (-3, 2, 1)


To Prove : Points A, B, C & D are coplanar.


To Find : Equation of plane passing through points A, B, C & D.


Formulae :


1) Position vectors :


If A is a point having co-ordinates (a1, a2, a3), then its position vector is given by,



2) Equation of line


If A and B are two points having position vectors then equation of line passing through two points is given by,



3) Cross Product :


If are two vectors




then,



4) Dot Product :


If are two vectors




then,



5) Coplanarity of two lines :


If two lines are coplanar then



6) Equation of plane :


If two lines are coplanar then equation of the plane containing them is



Where,



For given points,


A = (3, 2, -5)


B = (-1, 4, -3)


C = (-3, 8, -5)


D = (-3, 2, 1)


Position vectors are given by,






Equation of line passing through points A & B is






Let,


Where,


&


And the equation of the line passing through points C & D is






Let,


Where,


&


Now,





Therefore,



= 72 + 48 – 120


= 0


……… eq(1)


And



= - 72 + 192 – 120


= 0


……… eq(2)


From eq(1) and eq(2)



Hence lines are coplanar


Therefore, points A, B, C & D are also coplanar.


As lines are coplanar therefore equation of the plane passing through two lines containing four given points is



Now,



= 24x + 24y + 24z


From eq(1)



Therefore, equation of required plane is


24x + 24y + 24z = 0


x + y + z = 0



Question 3.

Show that the four points A(0, -1, 0),
B(2, 1, -1), C(1, 1, 1) and D(3, 3, 0) are coplanar. Find the equation of the plane containing them.


Answer:

Given Points :


A = (0, -1, 0)


B = (2, 1, -1)


C = (1, 1, 1)


D = (3, 3, 0)


To Prove : Points A, B, C & D are coplanar.


To Find : Equation of plane passing through points A, B, C & D.


Formulae :


1) Position vectors :


If A is a point having co-ordinates (a1, a2, a3), then its position vector is given by,



2) Equation of line


If A and B are two points having position vectors then equation of line passing through two points is given by,



3) Cross Product :


If are two vectors




then,



4) Dot Product :


If are two vectors




then,



5) Coplanarity of two lines :


If two lines are coplanar then



6) Equation of plane :


If two lines are coplanar then equation of the plane containing them is



Where,



For given points,


A = (0, -1, 0)


B = (2, 1, -1)


C = (1, 1, 1)


D = (3, 3, 0)


Position vectors are given by,






Equation of line passing through points A & D is






Let,


Where,


&


And equation of line passing through points B & C is






Let,


Where,


&


Now,





Therefore,



= 0 + 6 + 0


= 6


……… eq(1)


And



= 16 – 6 – 4


= 6


……… eq(2)


From eq(1) and eq(2)



Hence lines are coplanar


Therefore, points A, B, C & D are also coplanar.


As lines are coplanar therefore equation of the plane passing through two lines containing four given points is



Now,



= 8x - 6y + 4z


From eq(1)



Therefore, equation of required plane is


8x - 6y + 4z = 6


4x - 3y + 2z = 3



Question 4.

Write the equation of the plane whose intercepts on the coordinate axes are 2, -4 and 5 respectively.


Answer:

Given :


X – intercept, a = 2


Y – intercept, b = - 4


Z – intercept, c = 5


To Find : Equation of plane


Formula :


If a, b & c are the intercepts made by plane on X, Y & Z axes respectively, then equation of the plane is given by,




Multiplying above equation throughout by 40,



20x – 10y + 8z = 40


10x – 5y + 4z = 20


This the equation of the required plane.



Question 5.

Reduce the equation of the plane 4x – 3y + 2z = 12 to the intercept form, and hence find the intercepts made by the plane with the coordinate axes.


Answer:

Given :


Equation of plane : 4x – 3y + 2z = 12


To Find :


1) Equation of plane in intercept form


2) Intercepts made by the plane with the co-ordinate axes.


Formula :


If


is the equation of a plane in intercept form then intercept made by it with co-ordinate axes are


X-intercept = a


Y-intercept = b


Z-intercept = c


Given the equation of plane:


4x – 3y + 2z = 12


Dividing the above equation throughout by 12




This is the equation of a plane in intercept form.


Comparing the above equation with



We get,


a = 3


b = -4


c = 6


Therefore, intercepts made by plane with co-ordinate axes are


X-intercept = 3


Y-intercept = -4


Z-intercept = 6



Question 6.

Find the equation of the plane which passes through the point (2, -3,7) and makes equal intercepts on the coordinate axes.


Answer:

Equation of the plane making a, b & c intercepts with X, Y & Z axes respectively is



But, the plane makes equal intercepts on the co-ordinate axes


Therefore, a = b = c


Therefore the equation of the plane is



x + y + z = a


As plane passes through the point (2, -3, 7),


Substituting x = 2, y = -3 & z = 7


2 – 3 + 7 = a


Therefore, a = 6


Hence, required equation of plane is


x + y + z = 6



Question 7.

A plane meets the coordinate axes at A, B and C respectively such that the centroid of ∆ABC is (1, -2, 3). Find the equation of the plane.


Answer:

Given :


X-intercept = A


Y-intercept = B


Z-intercept = C


Centroid of ∆ABC = (1, -2, 3)


To Find : Equation of a plane


Formulae :


1) Centroid Formula :


For ∆ABC if co-ordinates of A, B & C are


A = (x1, x2, x3)


B = (y1, y2, y3)


C = (z1, z2, z3)


Then co-ordinates of the centroid of ∆ABC are



2) Equation of plane :


Equation of the plane making a, b & c intercepts with X, Y & Z axes respectively is



As the plane makes intercepts at points A, B & C on X, Y & Z axes respectively, let co-ordinates of A, B, C be


A = (a, 0, 0)


B = (0, b, 0)


C = (0, 0, c)


By centroid formula,


The centroid of ∆ABC is given by




But, Centroid of ∆ABC = (1, -2, 3) …… given



Therefore, a = 3, b = - 6, c = 9


Therefore,


X-intercept = a = 3


Y-intercept = b = - 6


Z-intercept = c = 9


Therefore, equation of required plane is





Question 8.

Find the Cartesian and vector equations of a plane passing through the point (1, 2, 3) and perpendicular to a line with direction ratios 2, 3, -4.


Answer:

Given :


A = (1, 2, 3)


Direction ratios of perpendicular vector = (2, 3, -4)


To Find : Equation of a plane


Formulae :


1) Position vectors :


If A is a point having co-ordinates (a1, a2, a3), then its position vector is given by,



2) Dot Product :


If are two vectors




then,



3) Equation of plane :


If a plane is passing through point A, then the equation of a plane is



Where,




For point A = (1, 2, 3), position vector is



Vector perpendicular to the plane with direction ratios (2, 3, -4) is



Now,


= 2 + 6 – 12


= - 4


Equation of the plane passing through point A and perpendicular to vector is




As



= 2x + 3y – 4z


Therefore, equation of the plane is


2x + 3y – 4z = -4


Or


2x + 3y – 4z + 4 = 0



Question 9.

If O is the origin and P(1, 2, -3) be a given point, then find the equation of the plane passing through P and perpendicular to OP.


Answer:

Given :


P = (1, 2, -3)


O = (0, 0, 0)



To Find : Equation of a plane


Formulae :


1) Position vectors :


If A is a point having co-ordinates (a1, a2, a3), then its position vector is given by,



2) Vector :


If A and B be two points with position vectors respectively, where




then,




3) Dot Product :


If are two vectors




then,



4) Equation of plane :


If a plane is passing through point A, then the equation of a plane is



Where,




For points,


P = (1, 2, -3)


O = (0, 0, 0)


Position vectors are




Vector





Now,



= 1 + 4 + 9


= 14


And



= x + 2y + 3z


Equation of the plane passing through point A and perpendicular to the vector is



But,


Therefore, the equation of the plane is



x + 2y + 3z = 14


x + 2y + 3z – 14 = 0




Exercise 28j
Question 1.

Find the direction ratios of the normal to the plane x + 2y - 3z = 5.


Answer:

Given :


Equation of plane : x + 2y – 3z = 5


To Find : direction ratios of normal


Answer :


Given equation of plane : x + 2y – 3z = 5


It can be written as



Comparing with


Therefore, normal vector is


Hence, direction ratios of normal are (1, 2, -3).



Question 2.

Find the direction cosines of the normal to the plane 2x + 3y - z = 4.


Answer:

Given :


Equation of plane : 2x + 3y – z = 4


To Find : Direction cosines of the normal i.e.


Formula :


1) Direction cosines :


If a, b & c are direction ratios of the vector then its direction cosines are given by





Answer :


For the given equation of plane


2x + 3y – z = 4


Direction ratios of normal vector are (2, 3, -1)





Therefore, direction cosines are







Question 3.

Find the direction cosines of the normal to the plane y = 3.


Answer:

Given :


Equation of plane : y = 3


To Find : Direction cosines of the normal i.e.


Formula :


1) Direction cosines :


If a, b & c are direction ratios of the vector then its direction cosines are given by





Answer :


For the given equation of plane


y = 3


Direction ratios of normal vector are (0, 1, 0)





= 1


Therefore, direction cosines are







Question 4.

Find the direction cosines of the normal to the plane 3x + 4 = 0.


Answer:

Given :


Equation of plane : 3x + 4 = 0


To Find : Direction cosines of the normal i.e.


Formula :


1) Direction cosines :


If a, b & c are direction ratios of the vector then its direction cosines are given by





Answer :


For the given equation of plane


-3x = 4


Direction ratios of normal vector are (-3, 0, 0)





= 3


Therefore, direction cosines are







Question 5.

Write the equation of the plane parallel to XY-plane and passing through the point (4, -2, 3).


Answer:

Given :


Point : (4, -2, 3)


To Find : equation of plane


Formula :


1) Equation of plane :


Equation of plane passing through point A with position vector and perpendicular to vector is given by,



Where,


Answer :


Position vector for given point A ≡ (4, -2, 3) is



As required plane is parallel to XY plane, therefore Z-axis is perpendicular to the plane.



Therefore, equation of plane is






This is required equation of plane.



Question 6.

Write the equation of the plane parallel to YZ-plane and passing through the point
(-3, 2, 0).


Answer:

Given :


Point : (-3, 2, 0)


To Find : equation of plane


Formula :


1) Equation of plane :


Equation of plane passing through point A with position vector and perpendicular to vector is given by,



Where,


Answer :


Position vector for given point A ≡ (-3, 2, 0) is



As required plane is parallel to YZ plane, therefore X-axis is perpendicular to the plane.



Therefore, equation of plane is






This is required equation of plane.



Question 7.

Write the general equation of a plane parallel to the x-axis.


Answer:

Let, normal vector of plane be



Equation of plane is given by,





As the required plane is parallel to the given plane, hence normal vector of plane is perpendicular to x-axis.




a = 0


Therefore, equation of plane is


by + cz = d



Question 8.

Write the intercept cut off by the plane 2x + y - z = 5 on the x-axis.


Answer:

Given :


Equation of plane : 2x + y – z = 5


To Find : Intercept made by the plane with the X-axis.


Formula :


If


is the equation of plane in intercept form then intercept made by it with co-ordinate axes are


X-intercept = a


Y-intercept = b


Z-intercept = c


Answer :


Given equation of plane:


2x + y – z = 5


Dividing above equation throughout by 5




Comparing above equation with



We get,


a = 5/2


Therefore, intercepts made by plane with X-axis are


X-intercept = 5/2



Question 9.

Write the intercepts made by the plane
4x - 3y + 2z = 12 on the coordinate axes.


Answer:

Given :


Equation of plane : 4x – 3y + 2z = 12


To Find :


1) Equation of plane in intercept form


2) Intercepts made by the plane with the co-ordinate axes.


Formula :


If


is the equation of plane in intercept form then intercept made by it with co-ordinate axes are


X-intercept = a


Y-intercept = b


Z-intercept = c


Answer :


Given equation of plane:


4x – 3y + 2z = 12


Dividing above equation throughout by 12




This is the equation of plane in intercept form.


Comparing above equation with



We get,


a = 3


b = -4


c = 6


Therefore, intercepts made by plane with co-ordinate axes are


X-intercept = 3


Y-intercept = -4


Z-intercept = 6



Question 10.

Reduce the equation 2x – 3y + 5z + 4 = 0 to intercept form and find the intercepts made by it on the coordinate axes.


Answer:

Given :


Equation of plane : 2x – 3y + 5z + 4 = 0


To Find :


1) Equation of plane in intercept form


2) Intercepts made by the plane with the co-ordinate axes.


Formula :


If


is the equation of plane in intercept form then intercept made by it with co-ordinate axes are


X-intercept = a


Y-intercept = b


Z-intercept = c


Answer :


Given equation of plane:


2x – 3y + 5z = -4


Dividing above equation throughout by -4




This is the equation of plane in intercept form.


Comparing above equation with



We get,


a = -2




Therefore, intercepts made by plane with co-ordinate axes are


X-intercept = -2





Question 11.

Find the equation of a plane passing through the points A(a, 0, 0), B(0, b, 0) and C(0, 0, c).


Answer:

Given : Plane is passing through points


A ≡ (a, 0, 0)


B ≡ (0, b, 0)


C ≡ (0, 0, c)


To Find : Equation of plane


Formulae :


Equation of plane making intercepts (a, b, c) on X, Y & Z axes respectively is given by,



Answer : As plane is passing through points A ≡ (a, 0, 0),


B ≡ (0, b, 0) & C ≡ (0, 0, c)


Therefore, intercepts made by it on X, Y & Z axes respectively are


a, b & c.


hence, equation of plane is




Question 12.

Write the value of k for which the planes 2x – 5y + kz = 4 and x + 2y – z = 6 are perpendicular to each other.


Answer:

Given : equations of perpendicular planes-


2x – 5y + kz = 4


x + 2y – z = 6


To Find : k


Formulae :


Normal vector to the plane :


If equation of the plane is ax + by + cz = d then,


Vector normal to the plane is given by,



Answer :


For given planes –


2x – 5y + kz = 4


x + 2y – z = 6


normal vectors are




As given vectors are perpendicular, hence their normal vectors are also perpendicular to each other.




(2×1) + (-5×2) + (k×(-1)) = 0


2 – 10 – k = 0


- 8 – k = 0


k = -8



Question 13.

Find the angle between the planes 2x + y – 2z = 5 and 3x – 6y – 2z = 7.


Answer:

Given : equations of planes-


2x + y – 2z = 5


3x – 6y – 2z = 7


To Find : angle between two planes


Formulae :


1) Normal vector to the plane :


If equation of the plane is ax + by + cz = d then,


Vector normal to the plane is given by,



2) Angle between two planes :


The angle Ө between the planes and is given by



Answer :


For given planes


2x + y – 2z = 5


3x – 6y – 2z = 7


Normal vectors are


and





Therefore, angle between two planes is









Question 14.

Find the angle between the planes and


Answer:

Given : equations of planes-




To Find : angle between two planes


Formulae :


Angle between two planes :


The angle Ө between the planes and is given by



Answer :


For given planes




Normal vectors are


and





Therefore, angle between two planes is










Question 15.

Find the angle between the planes and


Answer:

Given : equations of planes-




To Find : angle between two planes


Formulae :


Angle between two planes :


The angle Ө between the planes and is given by



Answer :


For given planes




Normal vectors are


and





Therefore, angle between two planes is










Question 16.

Find the angle between the line and the planes 10x + 2y – 11z = 3.


Answer:

Given :


Equation of line :


Equation of plane : 10x + 2y – 11z = 3


To Find : angle between line and plane


Formulae :


1) Parallel vector to the line :


If equation of the line is then,


Vector parallel to the line is given by,



2) Normal vector to the plane :


If equation of the plane is ax + by + cz = d then,


Vector normal to the plane is given by,



3) Angle between a line and a plane :


If Ө is a angle between the line and the plane , then



Where, is vector parallel to the line and


is the vector normal to the plane.


Answer :


For given equation of line,



Parallel vector to the line is




For given equation of plane,


10x + 2y – 11z = 3


normal vector to the plane is




Therefore, angle between given line and plane is










Question 17.

Find the angle between the line and the plane


Answer:

Given :


Equation of line :


Equation of plane :


To Find : angle between line and plane


Formulae :


1) Angle between a line and a plane :


If Ө is a angle between the line and the plane , then



Where, is vector parallel to the line and


is the vector normal to the plane.


Answer :


For given equation of line,



Parallel vector to the line is




For given equation of plane,



normal vector to the plane is




Therefore, angle between given line and plane is











Question 18.

Find the value of λ such that the line is perpendicular to the plane 3x - y – 2z = 7.


Answer:

Given :


Equation of line :


Equation of plane : 3x – y – 2z = 7


To Find :


Formulae :


1) Parallel vector to the line :


If equation of the line is then,


Vector parallel to the line is given by,



2) Normal vector to the plane :


If equation of the plane is ax + by + cz = d then,


Vector normal to the plane is given by,



3) Cross Product :


If are two vectors




then,



Answer :


For given equation of line,



Parallel vector to the line is



For given equation of plane,


3x – y – 2z = 7


normal vector to the plane is



As given line and plane are perpendicular to each other.





Comparing coefficients of on both sides



3λ = -6


λ = -2



Question 19.

Write the equation of the plane passing through the point (a, b, c) and parallel to the plane


Answer:

Given :


A ≡ (a, b, c)


Equation of plane parallel to required plane



To Find : Equation of plane


Formulae :


1) Position vectors :


If A is a point having co-ordinates (a1, a2, a3), then its position vector is given by,



2) Dot Product :


If are two vectors




then,



3) Equation of plane :


If a plane is passing through point A, then equation of plane is



Where,




Answer :


For point A ≡ (a, b, c), position vector is



As plane is parallel to the required plane, the vector normal to required plane is



Now,


= a + b + c


Equation of the plane passing through point A and perpendicular to vector is





Question 20.

Find the length of perpendicular drawn from the origin to the plane 2x – 3y + 6z + 21 = 0.


Answer:

Given :


Equation of plane : 2x – 3y + 6z + 21 = 0


To Find :


Length of perpendicular drawn from origin to the plane = d


Formulae :


1) Distance of the plane from the origin :


Distance of the plane from the origin is given by,



Answer :


For the given equation of plane


2x – 3y + 6z = -21


Direction ratios of normal vector are (2, -3, 6)


Therefore, equation of normal vector is






= 7


From given equation of plane,


p = -21


Now, distance of the plane from the origin is




d = 3 units



Question 21.

Find the direction cosines of the perpendicular from the origin to the plane


Answer:

Given :


Equation of plane :


To Find :


Direction cosines of the normal i.e.


Formulae :


1) Direction cosines :


If a, b & c are direction ratios of the vector then its direction cosines are given by





Answer :


For the given equation of plane



Equation of normal vector is






= 7


Therefore, direction cosines are







Question 22.

Show that the line is parallel to the plane


Answer:

Given :


Equation of plane : :


Equation of line :



To Prove : Given line is parallel to the given plane.


Answer :


Comparing given plane i.e.



with , we get,



This is the vector perpendicular to the given plane.


Now, comparing given equation of line i.e.



with , we get,



Now,




= 20 – 8 – 12


= 0



Therefore, vector normal to the plane is perpendicular to the vector parallel to the line.


Hence, the given line is parallel to the given plane.



Question 23.

Find the length of perpendicular from the origin to the plane


Answer:

Given :


Equation of plane :


To Find : Length of perpendicular = d


Formulae :


1) Unit Vector :


Let be any vector


Then unit vector of is



Where,


2) Length of perpendicular :


The length of the perpendicular from the origin to the plane


is given by,



Answer :


Given equation of the plane is





Comparing above equation with



We get,


& p = 14


Therefore,





= 7


The length of the perpendicular from the origin to the given plane is






Question 24.

Find the value of λ for which the line

is parallel to the plane


Answer:

Given :


Equation of line :


Equation of plane :


To Find : λ


Formulae :


1) Parallel vector to the line :


If equation of the line is then,


Vector parallel to the line is given by,



2) Angle between a line and a plane :


If Ө is a angle between the line and the plane , then



Where, is vector parallel to the line and


is the vector normal to the plane.


Answer :


For given equation of line,



Parallel vector to the line is



For given equation of plane,



normal vector to the plane is



Therefore, angle between given line and plane is



As given line is parallel too the given plane, angle between them is 0.







4 + 9 + 4 λ = 0


13 + 4λ = 0


4λ = -13





Question 25.

Write the angle between the line

and the plane x + y + 4 = 0.


Answer:

Given :


Equation of line :


Equation of plane : x + y + 4 = 0


To Find : angle between line and plane


Formulae :


1) Parallel vector to the line :


If equation of the line is then,


Vector parallel to the line is given by,



2) Normal vector to the plane :


If equation of the plane is ax + by + cz = d then,


Vector normal to the plane is given by,



3) Angle between a line and a plane :


If Ө is a angle between the line and the plane , then



Where, is vector parallel to the line and


is the vector normal to the plane.


Answer :


For given equation of line,



Parallel vector to the line is




For given equation of plane,


x + y + 4 = 0


normal vector to the plane is




Therefore, angle between given line and plane is











Question 26.

Write the equation of a plane passing through the point (2, -1, 1) and parallel to the plane 3x + 2y - z = 7.


Answer:

Given :


A ≡ (2, -1, 1)


Plane parallel to the required plane : 3x + 2y – z = 7


To Find : Equation of plane


Formulae :


1) Position vectors :


If A is a point having co-ordinates (a1, a2, a3), then its position vector is given by,



2) Dot Product :


If are two vectors




then,



3) Equation of plane :


If a plane is passing through point A, then equation of plane is



Where,




Answer :


For point A ≡ (2, -1, 1), position vector is



As required plane is parallel to 3x + 2y – z = 7.


Therefore, normal vector of given plane is also perpendicular to required plane



Now,


= 6 – 2 – 1


= 3


Equation of the plane passing through point A and perpendicular to vector is




As



= 3x + 2y – z


Therefore, equation of the plane is


3x + 2y – z = 3


3x + 2y – z - 3 = 0




Objective Questions
Question 1.

Mark against the correct answer in each of the following:

The direction cosines of the perpendicular from the origin to the plane are

A.

B.

C.

D. None of these


Answer:

Given: Equation of plane is


Formula Used: Equation of a plane is where is the unit vector normal to the plane, represents a point on the plane and is the distance of the plane from the origin.


Explanation:


The equation of the given plane is … (1)


Now,


= 7


is a unit vector.


(1) can be rewritten as




which is of the form


Perpendicular vector from the origin to the plane is



So, direction cosines of the vector perpendicular from the origin to the plane is


Question 2.

Mark against the correct answer in each of the following:

The direction cosines of the normal to the plane 5y + 4 = 0 are

A.

B. 0, 1, 0

C. 0, -1, 0

D. None of these


Answer:

Given: Equation of plane is 5y + 4 = 0


Formula Used: Equation of a plane is lx + my + nz = p where (l, m, n) are the direction cosines of the normal to the plane and (x, y, z) is a point on the plane and p is the distance of plane from origin.


Explanation:


Given equation is 5y = -4


Dividing by -5,



which is of the form lx + my + nz = p where l = 0, m = -1, n = 0


Therefore, direction cosines of the normal to the plane is (0, -1, 0)


Question 3.

Mark against the correct answer in each of the following:

The length of perpendicular from the origin to the plane is

A. 3 units

B. units

C. units

D. None of these


Answer:

Given: Equation of plane is


Formula Used: Equation of a plane is where is the unit vector normal to the plane, represents a point on the plane and is the distance of the plane from the origin.


Explanation:


Given equation is … (1)


Now,


= 13


Dividing (1) by 13 and multiplying by -1,



which is of the form


Therefore, length of perpendicular from origin to plane is 3 units.


Question 4.

Mark against the correct answer in each of the following:

The equation of a plane passing through the point A(2, -3, 7) and making equal intercepts on the axes, is

A. x + y + z = 3

B. x + y + z = 6

C. x + y + z = 9

D. x + y + z = 4


Answer:

Given: A(2, -3, 7) is a point on the plane making equal intercepts on the axes.


Formula Used: Equation of plane is where (x, y, z) is a point on the plane and a, b, c are intercepts on x-axis, y-axis and z-axis respectively.


Explanation:


Let the equation of the plane be


… (1)


Here a = b = c = p (let’s say)


Since (2, -3, 7) is a point on the plane,


(1) becomes



p = 6


Therefore equation of the plane is


x + y + z = 6


Question 5.

Mark against the correct answer in each of the following:

A plane cuts off intercepts 3, -4, 6 on the coordinate axes. The length of perpendicular from the origin to this plane is

A. units

B. units

C. units

D. units


Answer:

Given: Plane makes intercepts 3, -4 and 6 with the coordinate axes.


Formula Used: Equation of plane is where (x, y, z) is a point on the plane and a, b, c are intercepts on x-axis, y-axis and z-axis respectively.


Normal Form of a plane ⇒ lx + my + nz = p where (l, m, n) is the direction cosines and p is the distance of perpendicular to the plane from the origin.


Explanation:


Equation of the given plane is



i.e., 4x – 3y + 2z = 12 … (1)


which is of the form ax + by + cz = d


Direction ratios are (4, -3, 12)


So,


= √29


Dividing (1) by 13,



which is in the normal form


Therefore length of perpendicular from the origin is units


Question 6.

Mark against the correct answer in each of the following:

If the line is parallel to the plane 2x – 3y + kz = 0, then the value of k is

A.

B.

C.

D.


Answer:

Given:


1. Equation of line is


2. Equation of plane is 2x – 3y + kz = 0


Formula Used: If two direction ratios are perpendicular, then


a �1a2 + b1b2 + c1c2 = 0


Explanation:


Direction ratios of given line is (3, 4, 5)


Direction ratios of given plane is (2, -3, k)


Since the given line is parallel to the plane, the normal to the plane is perpendicular to the line.


So direction ratio of line is perpendicular to direction ratios of plane.


⇒ 3× 2 + 4× -3+ 5× k = 0


⇒ 6 – 12 + 5k = 0



Therefore,


Question 7.

Mark against the correct answer in each of the following:

If O is the origin and P(1, 2, -3) is a given point, then the equation of the plane through P and perpendicular to OP is

A. x + 2y – 3z = 14

B. x – 2y + 3z = 12

C. x – 2y – 3z = 14

D. None of these


Answer:

Given: P(1, 2, -3) is a point on the plane. OP is perpendicular to the plane.


Explanation:


Let equation of plane be ax + by + cz = d … (1)


Substituting point P,


⇒ a + 2b -3c = d … (2)



Since OP is perpendicular to the plane, direction ratio of the normal is (1, 2, -3)


Substituting in (2)


1 + 4 + 9 = d


d = 14


Substituting the direction ratios and value of ‘d’ in (1), we get


x + 2y – 3z = 14


Therefore equation of plane is x + 2y – 3z = 14


Question 8.

Mark against the correct answer in each of the following:

If the line lies in the plane 2x – 4y + z = 7, then the value of k is

A. -7

B. 7

C. 4

D. -4


Answer:

Given: Equation of plane is 2x – 4y + z = 7


Line lies on the given plane.


Formula Used: Equation of a line is



Where (x1, y1, z1) is a point on the line and b1, b2, b3 : direction ratios of line.


Explanation:


Let


So the given line passes through the point (4, 2, k)


Since the line lies on the given plane, (4, 2, k) is a point on the plane.


Therefore, substituting the point on the equation for the plane,


⇒ 8 – 8 + k = 7


⇒ k = 7


Question 9.

Mark against the correct answer in each of the following:

The plane 2x + 3y + 4z =12 meets the coordinate axes in A, B and C. The centroid of ∆ABC is

A. (2, 3, 4)

B. (6, 4, 3)

C.

D. None of these


Answer:

Given: The plane 2x + 3y + 4z = 12 meets coordinate axes at A, B and C.


To find: Centroid of ∆ABC


Formula Used: Equation of plane is where (x, y, z) is a point on the plane and a, b, c are intercepts on x-axis, y-axis and z-axis respectively.


Centroid of a triangle


Explanation:


Equation of given plane is 2x + 3y + 4z = 12


Dividing by 12,



Therefore the intercepts on x, y and z-axis are 6, 6 and 3 respectively.


So, the vertices of ∆ABC are (6, 0, 0), (0, 4, 0) and (0, 0, 3)


Centroid


= (2, 4/3, 1)


Therefore, the centroid of ∆ABC is (2, 4/3, 1)


Question 10.

Mark against the correct answer in each of the following:

If a plane meets the coordinate axes in A, B and C such that the centroid of ∆ABC is (1, 2, 4), then the equation of the plane is

A. x + 2y + 4z = 6

B. 4x + 2y + z = 12

C. x + 2y + 4z = 7

D. 4x + 2y + z = 7


Answer:

Given: Centroid of ∆ABC is (1, 2, 4)


To find: Equation of plane.


Formula Used: Equation of plane is where (x, y, z) is a point on the plane and a, b, c are intercepts on x-axis, y-axis and z-axis respectively.


Centroid of a triangle


Explanation:


Let the equation of plane be


… (1)


Therefore, A = 3a, B = 3b, C = 3c where (a, b, c) is the centroid of the triangle with vertices (A, 0, 0), (0, B, 0) and (0, 0, C)


Substituting in (1),



Here a = 1, b = 2 and c = 4



Multiplying by 12,


4x + 2y + z = 12


Therefore equation of required plane is 4x + 2y + z = 12


Question 11.

Mark against the correct answer in each of the following:

The equation of a plane through the point A(1, 0, -1) and perpendicular to the line is

A. 2x + 4y – 3z = 3

B. 2x – 4y + 3z = 5

C. 2x + 4y – 3z = 5

D. x + 3y + 7z = -6


Answer:

Given: Plane passes through the point A(1, 0, -1).


Plane is perpendicular to the line



To find: Equation of the plane.


Formula Used: Equation of a plane is ax + by + cz = d where (a, b, c) are the direction ratios of the normal to the plane.


Explanation:


Let the equation of the plane be


ax + by + cz = d … (1)


Substituting point A,


a – z = d


Since the given line is perpendicular to the plane, it is the normal.


Direction ratios of line is 2, 4, -3


Therefore, 2 + 3 = d


d = 5


So the direction ratios of perpendicular to plane is 2, 4, -3 and d = 5


Substituting in (1),


2x + 4y – 3z = 5


Therefore, equation of plane is 2x + 4y – 3z = 5


Question 12.

Mark against the correct answer in each of the following:

The line meets the plane 2x + 3y – z = 14 in the point

A. (2, 5, 7)

B. (3, 5, 7)

C. (5, 7, 3)

D. (6, 5, 3)


Answer:

Given: Line meets plane 2x + 3y – z = 14


To find: Point of intersection of line and plane.


Explanation:


Let the equation of the line be



Therefore, any point on the line is (2λ + 1, 4λ +2, -3λ +3)


Since this point also lies on the plane,


2(2λ + 1) + 3(4λ +2) – (-3λ +3) =14


4λ + 2 + 12λ + 6 + 3λ – 3 = 14


19λ + 5 = 14



Therefore the required point is (3, 5, 7).


Question 13.

Mark against the correct answer in each of the following:

The equation of the plane passing through the points A(2, 2, 1) and B(9, 3, 6) and perpendicular to the plane 2x + 6y + 6z = 1, is

A. x + 2y – 3z + 5 = 0

B. 2x – 3y + 4z – 6 = 0

C. 4x + 5y – 6z + 3 = 0

D. 3x + 4y – 5z – 9 = 0


Answer:

Given: Plane passes through A(2, 2, 1) and B(9, 3, 6). Plane is perpendicular to 2x + 6y + 6z = 1


To find: Equation of the plane


Formula Used: Equation of a plane is


a(x – x1) + b(y – y1) + c(z – z1) = 0


where a:b:c is the direction ratios of the normal to the plane.


(x1, y1, z1) is a point on the plane.


Explanation:


Let the equation of plane be a(x – x1) + b(y – y1) + c(z – z1) = 0


Since (2, 2, 1) is a point in the plane,


a(x – 2) + b(y – 2) + c(z – 1) = 0 … (1)


Since B(9, 3, 6) is another point on the plane,


a(9 – 2) + b(3 – 2) + c(6 – 1) = 0


7a + b + 5c = 0 … (1)


Since this plane is perpendicular to the plane 2x + 6y + 6z = 1, the direction ratios of the normal to the plane will also be perpendicular.


So, 2a + 6b + 6c = 0 ⇒ a + 3b + 3c = 0 … (2)


Solving (1) and (2),





a : b : c = 3 : 4 : -5


Substituting in (1),


3x – 6 + 4y – 8 – 5z + 5 = 0


3x + 4y – 5z – 9 = 0


Therefore the equation of the plane is 3x + 4y – 5z – 9 = 0


Question 14.

Mark against the correct answer in each of the following:

The equation of the plane passing through the intersection of the planes 3x - y + 2z – 4 = 0 and x + y + z - 2 = 0 and passing through the point A(2, 2, 1) is given by

A. 7x + 5y – 4z – 8 = 0

B. 7x – 5y + 4z – 8 = 0

C. 5x – 7y + 4z – 8 = 0

D. 5x + 7y – 4z + 8 = 0


Answer:

Given: Plane passes through the intersection of planes 3x – y + 2z – 4 = 0 and x + y + z – 2 = 0. Point A(2, 2, 1) lies on the plane.


To find: Equation of the plane.


Formula Used: Equation of plane passing through the intersection of 2 planes P �1 and P2 is given by P1 + λP2 = 0


Explanation:


Equation of plane is


3x – y + 2z – 4 + λ (x + y + z - 2) = 0 … (1)


Since A(2, 2, 1) lies on the plane,


6 – 2 + 2 – 4 + λ (2 + 2 + 1 – 2) = 0


2 + 3λ = 0



Substituting in (1) and multiplying by 3,


9x – 3y + 6z – 12 – 2 (x + y + z - 2) = 0


9x – 3y + 6z – 12 – 2x – 2y – 2z + 4 = 0


7x – 5y + 4z – 8 = 0


Therefore the equation of the plane is 7x – 5y + 4z – 8 = 0


Question 15.

Mark against the correct answer in each of the following:

The equation of the plane passing through the points A(0, -1, 0), B(2, 1, -1) and C(1, 1, 1) is given by

A. 4x + 3y – 2z – 3 = 0

B. 4x – 3y + 2z + 3 = 0

C. 4x – 3y + 2z – 3 = 0

D. None of these


Answer:

Given: Plane passes through A(0, -1, 0), B(2, 1, -1) and C(1, 1, 1)


To find: Equation of the plane


Formula Used: Equation of a plane is


a(x – x1) + b(y – y1) + c(z – z1) = 0


where a:b:c is the direction ratios of the normal to the plane.


(x1, y1, z1) is a point on the plane.


Explanation:


Let the equation of plane be a(x – x1) + b(y – y1) + c(z – z1) = 0


Substituting point A,


ax + b(y + 1) + cz = 0 … (1)


Substituting points B and C,


2a + 2b – c = 0 and a + 2b +c = 0


Solving,




Therefore, a : b : c = 4 : -3 : 2


Substituting in (1),


4x – 3 (y + 1) + 2z = 0


4x – 3y + 2z – 3 = 0


Therefore equation of plane is 4x – 3y + 2z – 3 = 0


Question 16.

Mark against the correct answer in each of the following:

If the plane 2x – y + z = 0 is parallel to the line then the value of a is

A. -4

B. -2

C. 4

D. 2


Answer:

Given: Plane 2x – y + z = 0 is parallel to the line



To find: value of a


Formula Used: If two lines with direction ratios a1:a2:a3 and b1:b2:b3 are perpendicular, then


a1b1 + a2b2 + a3b3 = 0


Explanation:


Since the plane is parallel to the line, the normal to the plane will be perpendicular to the line.


Equation of the line can be rewritten as



Direction ratio of the normal to the plane is 2 : -1 : 1


Direction ratio of line is 1 : -2 : a


Therefore,


2 + 2 + a = 0


a = -4


Therefore, a = -4


Question 17.

Mark against the correct answer in each of the following:

The angle between the line and a normal to the plane x – y + z = 0 is

A. 0°

B. 30°

C. 45°

D. 90°


Answer:

Given: Equation of line is


Equation of plane is x – y + z = 0


To find: Angle between a line and the normal to a plane.


Formula Used: If θ is the angle between two lines with direction ratios b1:b2:b3 and c1:c2:c3, then



Explanation:


Direction ratios of given line is 1 : 2 : 1


Direction ratios of the normal to the plane is 1 : -1 : 1


Therefore,



cos θ = 0


θ = 90°


Therefore angle between them is 90°


Question 18.

Mark against the correct answer in each of the following:

The point of intersection of the line and the plane 2x – y + 3z – 1 = 0, is

A. (-10, 10, 3)

B. (10, 10, -3)

C. (10, -10, 3)

D. (10, -10, -3)


Answer:

Given: Line meets plane 2x – y + 3z – 1 = 0


To find: Point of intersection of line and plane.


Explanation:


Let the equation of the line be



Therefore, any point on the line is (3λ + 1, 4λ - 2, -2λ + 3)


Since this point also lies on the plane,


2(3λ + 1) - (4λ -2) + 3(-2λ +3) = 1


6λ + 2 – 4λ + 2 – 6λ + 9 = 1


-4λ = -12


λ = 3


Therefore required point is (10, 10, -3)


Question 19.

Mark against the correct answer in each of the following:

The equation of a plane passing through the points A(a, 0, 0), B(0, b, 0) and C(0, 0, c) is given by

A. ax + by + cz = 0

B. ax + by + cz = 1

C.

D.


Answer:

Given: Plane passes through the points A(a, 0, 0), B(0, b, 0) and C(0, 0, c)


To find: Equation of plane.


Explanation:


The given points lie on the co-ordinate axes.


Therefore, the plane makes intercepts of a, b and c on the x, y and z-axis respectively.


Equation of the plane is



Question 20.

Mark against the correct answer in each of the following:

If θ is the angle between the planes 2x – y + 2z = 3 and 6x – 2y + 3z = 5, then cos θ = ?

A.

B.

C.

D.


Answer:

Given: Equation of two planes are 2x – y + 2z = 3 and 6x – 2y + 3z = 5


To find: cos θ where θ: angle between the planes


Formula Used: Angle between two planes a1x + b1y + c1z = 0 and a2x + b2y + c2z = 0 is



where θ : angle between the planes,


Explanation:


Here a1 = 2, b1 = -1, c1 = 2


a2 = 6, b2 = -2, c2 = 3





Therefore,


Question 21.

Mark against the correct answer in each of the following:

The angle between the planes 2x – y + z = 6 and x + y + 2z = 7, is

A.

B.

C.

D.


Answer:

Given: Equation of two planes are 2x – y + z = 6 and x + y + 2z = 7


To find: Angle between the two planes


Formula Used: Angle between two planes a1x + b1y + c1z = 0 and a2x + b2y + c2z = 0 is



where θ : angle between the planes,


Explanation:


Here a1 = 2, b1 = -1, c1 = 1


a2 = 1, b2 = 1, c2 = 2







Therefore angle between the planes is


Question 22.

Mark against the correct answer in each of the following:

The angle between the planes and is

A.

B.

C.

D.


Answer:

Given: Equation of two planes are and


To find: Angle between the two planes


Formula Used: Angle between two planes a1x + b1y + c1z = 0 and a2x + b2y + c2z = 0 is



where θ : angle between the planes,


Explanation:


Since , the given equation of planes can rewritten as:


3x – 6y + 2z = 4 and 2x – y + 2z = 3


Here a1 = 3, b1 = -6, c1 = 2


a2 = 2, b2 = -1, c2 = 2






Therefore angle between the planes is


Question 23.

Mark against the correct answer in each of the following:

The equation of the plane through the points A(2, 3, 1) and B(4, -5, 3), parallel to the x-axis, is

A. x + y – 3z = 2

B. y + 4z = 7

C. y + 3z = 6

D. x + 5y – 3z = 4


Answer:

Given: Plane passes through the points A(2, 3, 1) and B(4, -5, 3) and is parallel to x-axis


To find: Equation of plane


Formula Used: Equation of a plane parallel o x-axis is


b(y – y1) + c(z – z1) = 0


Explanation:


Let the equation of the plane be


b(y – y1) + c(z – z1) = 0


Since A(2, 3, 1) lies on the plane,


b(y – 3) + c(z – 1) = 0 … (1)


Since B(4, -5, 3) lies on the plane,


b(-5 – 3) + c(3 – 1) = 0


-8b + 2c = 0 or –4b + c = 0


b : c = 1 : 4


Substituting in (1),


y – 3 + 4z – 4 = 0


y + 4z = 7


The equation of the plane is y + 4z = 7


Question 24.

Mark against the correct answer in each of the following:

A variable plane moves so that the sum of the reciprocals of its intercepts on the coordinate axes is (1/2). Then, the plane passes through the point

A. (0, 0, 0)

B. (1, 1, 1)

C.

D. (2, 2, 2)


Answer:

Given: Variable plane moves so that the sum of the reciprocals of its intercepts on the coordinate axes is (1/2)


Formula Used: Equation of a plane is



Explanation:


Let the intercepts made by the plane on the co-ordinate axes be a, b and c.



Let the equation of the plane be



On solving for each of the given options,


(0, 0, 0) ⇒ LHS ≠ RHS


(1, 1, 1) ⇒ LHS ≠ RHS




Therefore, plane passes through the point (2, 2, 2)


Question 25.

Mark against the correct answer in each of the following:

The equation of a plane which is perpendicular to and at a distance of 5 units from the origin is

A. 2x – 3y + z = 5

B. 2x – 3y + z = 5√14

C.

D.


Answer:

Given: Plane is perpendicular to and is at a distance of 5 units from origin.


To find: Equation of plane


Formula Used: Equation of a plane is lx + my + nz = p where p is the distance from the origin and l, m and n are the direction cosines of the normal to the plane


Explanation:


Direction ratio of normal to the plane is 2:-3:1



Therefore, direction cosines of the normal to the plane is


l , m , n


Since the equation of a plane is lx + my + nz = p where p is the distance from the origin,


2x – 3y + z = 5√14


Therefore, equation of the plane is 2x – 3y + z = 5√14


Question 26.

Mark against the correct answer in each of the following:

The equation of the plane passing through the point A(2, 3,4) and parallel to the plane 5x - 6y + 7z = 3, is

A. 5x – 6y + 7z = 20

B. 7x – 6y + 5z = 72

C. 20x – 18y + 14z = 11

D. 10x – 18y + 28z = 13


Answer:

Given: Point A(2, 3, 4) lies on a plane which is parallel to 5x – 6y + 7z = 3


To find: Equation of the plane


Formula Used: Equation of a plane is


a(x – x1) + b(y – y1) + c(z – z1) = 0


where a:b:c is the direction ratios of the normal to the plane


(x1, y1, z1) is a point on the plane.


Explanation:


Since the plane (say P1) is parallel to the plane 5x – 6y + 7z = 3 (say P2), the direction ratios of the normal to P1 is same as the direction ratios of the normal to P2.


i.e., direction ratios of P1 is 5 : -6 : 7


Let the equation of the required plane be


a(x – x1) + b(y – y1) + c(z – z1) = 0


Here a = 5, b = -6 and c = 7


Since (2, 3, 4) lies on the plane,


5(x - 2) – 6 (y - 3) + 7 (z - 4) = 0


5x – 6y + 7z – 10 + 18 – 28 = 0


5x – 6y + 7z = 20


The equation of the plane is 5x – 6y + 7z = 20


Question 27.

Mark against the correct answer in each of the following:

The foot of the perpendicular from the point A(7, 14, 5) to the plane 2x + 4y – z = 2 is

A. (3, 1, 8)

B. (1, 2, 8)

C. (3, -3, 5)

D. (5, -3, -4)


Answer:

Given: Perpendicular dropped from A(7, 14, 5) on to the plane 2x + 4y – z = 2


To find: co-ordinates of the foot of perpendicular


Formula Used: Equation of a line is



Where b1:b2:b3 is the direction ratio and (x1, x2, x3) is a point on the line.


Explanation:


Let the foot of the perpendicular be (a, b, c)


Since this point lies on the plane,


2a + 4b – c = 2 … (1)


Direction ratio of the normal to the plane is 2 : 4 : -1


Direction ratio perpendicular = direction ratio of normal to the plane


So, equation of the perpendicular is



Since (a, b, c) is a point on the perpendicular,



(7, 14, 5) is a point on the perpendicular.



So, a = 7 - 2λ, b = 14 – 4λ, c = 5 + λ


Substituting in (1),


14 – 4λ + 56 – 16λ – 5 - λ = 2


21λ = 70 – 7 = 63


λ = 3


Therefore, foot of the perpendicular is (1, 2, 8)


Question 28.

Mark against the correct answer in each of the following:

The equation of the plane which makes with the coordinate axes, a triangle with centroid (α, β, γ) is given by

A. αx + βy + γz = 1

B. αx + βy + γz = 3

C.

D.


Answer:

Given: Centroid of triangle is (α, β, γ)


To find: Equation of plane.


Formula Used: Equation of plane is where (x, y, z) is a point on the plane and a, b, c are intercepts on x-axis, y-axis and z-axis respectively.


Centroid of a triangle


Explanation:


Let the equation of plane be


… (1)


Therefore, A = 3α, B = 3β, C = 3γ where (a, b, c) is the centroid of the triangle with vertices (A, 0, 0), (0, B, 0) and (0, 0, C)


Substituting in (1),



Here a = α, b = β and c = γ



Therefore equation of required plane is


Question 29.

Mark against the correct answer in each of the following:

The intercepts made by the plane are

A. 2, -3, 4

B. 2, -3, -6

C. 6, -4, 3

D. -6, 4, 3


Answer:

Given: Equation of plane is


To find: Intercepts made by the plane.


Formula Used: Equation of plane is where (x, y, z) is a point on the plane and a, b, c are intercepts on x-axis, y-axis and z-axis respectively.


Explanation:


The equation of the plane can be written as


2x – 3y + 4z = 12


Dividing by 12,


which is of the form


Therefore the intercepts made by the plane are 6, -4, 3


Question 30.

Mark against the correct answer in each of the following:

The angle between the line and the plane 2x – 3y + z = 5 is

A.

B.

C.

D.


Answer:

Given: Equation of line is


Equation of the plane is 2x – 3y + z = 5


To find: angle between line and plane


Formula Used: If θ is the angle between a line with direction ratio b1:b2:b3 and a plane with direction ratio of normal n1:n2:n3, then



Explanation:


Here direction ratio of the line is 1 : -2 : -3


Direction ratio of normal to the plane is 2 : -3 : 1


Therefore,





Therefore, angle between the line and plane is


Question 31.

Mark against the correct answer in each of the following:

The angle between the line and the plane is

A.

B.

C.

D.


Answer:

Given: Equation of line is


Equation of plane is


To find: angle between line and plane


Formula Used: If θ is the angle between a line with direction ratio b1:b2:b3 and a plane with direction ratio of normal n1:n2:n3, then



Explanation:


Here direction ratio of the line is 2 : 2 : 1


Direction ratio of normal to the plane is 6 : -3 : 2


Therefore,





Therefore, angle between the line and plane is


Question 32.

Mark against the correct answer in each of the following:

The distance of the point from the plane is

A. units

B. units

C. 25√2 units

D. 25√3 units


Answer:

Given: Point is at and equation of plane is


To find: distance of point from plane


Formula Used: Perpendicular distance from (x1, y1, z1) to the plane ax + by + cz + d = 0 is



Explanation:


The point is at (1, 2, 5) and equation of plane is x + y + z + 17 = 0


Distance



Therefore, distance units


Question 33.

Mark against the correct answer in each of the following:

The distance between the parallel planes 2x – 3y + 6z = 5 and 6x – 9y + 18z + 20 = 0, is

A. units

B. 5√3 units

C. units

D. 8√5 units


Answer:

Given: The equations of the parallel planes are 2x – 3y + 6z = 5 and 6x – 9y + 18z + 20 = 0


To find: distance between the planes


Formula Used: Distance between two parallel planes ax + by + cz + d1 = 0 and ax1 + by1 + cz1 + d1 = 0 is



Explanation:


The equations of the parallel planes are:


2x – 3y + 6z – 5 = 0



Therefore distance between them is





Therefore distance between the planes is units


Question 34.

Mark against the correct answer in each of the following:

The distance between the planes x + 2y – 2z + 1 = 0 and 2x + 4y – 4z – 4z + 5 = 0, is

A. 4 units

B. 2 units

C. units

D. units


Answer:

Given: The equations of the planes are x + 2y – 2z + 1 = 0 and 2x + 4y – 4z – 4z + 5 = 0


To find: distance between the planes


Formula Used: Distance between two parallel planes ax + by + cz + d1 = 0 and ax1 + by1 + cz1 + d1 = 0 is



Explanation:


The equations of the planes are:


x + 2y – 2z + 1 = 0 and 2x + 4y – 4z – 4z + 5 = 0


Multiplying the equation of first plane by 2,


2x + 4y – 4z + 2 = 0


Therefore distance between them is





Therefore distance between the planes is units


Question 35.

Mark against the correct answer in each of the following:

The image of the point P(1, 3, 4) in the plane 2x – y + z + 3 = 0, is

A. (3, -5, 2)

B. (3, 5, -2)

C. (3, 5, 2)

D. (-3, 5, 2)


Answer:

Given: Equation of plane is 2x – y + z + 3 = 0. P is at (1, 3, 4)


To find: image of P


Explanation:



From the figure, P’ is the image of P and B is the midpoint of PP’


If B is (a, b, c), then





B lies on the plane.


So, 2a – b + c + 3 = 0 … (1)


Also, equation of line PB is



So any point on the line PB will be of the form


x = 2λ +1, y = -λ + 3, z = λ + 4


Since (a, b, c) will also be of this form we can substitute these values in (1)


⇒ 2(2λ + 1) – (-λ + 3) + (λ + 4) + 3 = 0


⇒ 4λ + 2 + λ – 3 + λ + 4 + 3 = 0


⇒ 6λ = -6


⇒ λ = -1


So (a, b, c) = (-1, 4, 3)


Substituting these values in the equations of a, b and c,





Therefore,


x1 = -3, y1 = 5, z1 = 2


Therefore, the image is (-3, 5, 2)



Exercise 28b
Question 1.

Find the vector and Cartesian equations of a plane which is at a distance of 5 units from the origin and which has as the unit vector normal to it.


Answer:

Given :


d = 5



To Find : Equation of a plane


Formulae :


1) Dot Product :


If are two vectors




then,



2) Equation of plane :


Equation of plane which is at a distance of 5 units from the origin and having as a unit vector normal to it is



Where,


For given d = 5 and ,


Equation of plane is




This is a vector equation of the plane


Now,




= z



Therefore, the equation of the plane is


This is - the Cartesian z = 5 equation of the plane.



Question 2.

Find the vector and Cartesian equations of a plane which is at a distance of 7 units from the origin and whose normal vector from the origin is


Answer:

Given :


d = 7



To Find : Equation of plane


Formulae :


1) Unit Vector :


Let be any vector


Then unit vector of is



Where,


2) Dot Product :


If are two vectors




then,



3) Equation of plane :


Equation of plane which is at a distance of 5 units from the origin and having as a unit vector normal to it is



Where,


For given normal vector



Unit vector normal to the plane is






Equation of the plane is





This is a vector equation of the plane.


Now,



= (x × 3) + (y × 5) + (z × (-6))


= 3x + 5y – 6z


Therefore equation of the plane is



This is the Cartesian equation of the plane.



Question 3.

Find the vector and Cartesian equations of a plane which is at a distance of from the origin and whose normal vector from the origin is


Answer:

Given :




To Find : Equation of a plane


Formulae :


1) Unit Vector :


Let be any vector


Then the unit vector of is



Where,


2) Dot Product :


If are two vectors




then,



3) Equation of plane :


Equation of plane which is at a distance of 5 units from the origin and having as a unit vector normal to it is



Where,


For given normal vector



Unit vector normal to the plane is






Equation of the plane is





This is a vector equation of the plane.


Now,



= (x × 2) + (y × (-3)) + (z × 4)


= 2x - 3y + 4z


Therefore equation of the plane is


2x - 3y + 4z = 6


This is the Cartesian equation of the plane.



Question 4.

Find the vector and Cartesian equations of a plane which is at a distance of 6 units from the origin and which has a normal with direction ratios 2, -1, -2.


Answer:

Given :


d = 6


direction ratios of are (2, -1, -2)



To Find : Equation of plane


Formulae :


1) Unit Vector :


Let be any vector


Then the unit vector of is



Where,


2) Dot Product :


If are two vectors




then,



3) Equation of plane :


Equation of plane which is at a distance of 5 units from the origin and having as a unit vector normal to it is



Where,


For given normal vector



Unit vector normal to the plane is







Equation of the plane is





This is vector equation of the plane.


Now,



= (x × 2) + (y × (-1)) + (z × (-2))


= 2x - y – 2z


Therefore equation of the plane is



This is Cartesian equation of the plane.



Question 5.

Find the vector, and Cartesian equations of a plane which passes through the point (1, 4, 6) and the normal vector to the plane is


Answer:

Given :


A = (1, 4, 6)



To Find : Equation of plane.


Formulae :


1) Position Vector :


If A is a point having co-ordinates (a1, a2, a3), then its position vector is given by,



2) Dot Product :


If are two vectors




then,



3) Equation of plane :


Equation of plane passing through point A and having as a unit vector normal to it is



Where,


Position vector of point A = (1, 4, 6) is



Now,



= (1×1) + (4×(-2)) + (6×1)


= 1 – 8 + 6


= - 1


Equation of plane is




This is vector equation of the plane.


As


Therefore



= (x × 1) + (y × (-2)) + (z × 1)


= x - 2y + z


Therefore equation of the plane is



This is Cartesian equation of the plane.



Question 6.

Find the length of the perpendicular from the origin to the plane Also write the unit normal vector from the origin to the plane.


Answer:

Given :


Equation of plane :


To Find :


i) Length of perpendicular = d


ii) Unit normal vector =


Formulae :


1) Unit Vector :


Let be any vector


Then unit vector of is



Where,


2) Length of perpendicular :


The length of the perpendicular from the origin to the plane


is given by,



Given the equation of the plane is





Comparing the above equation with



We get,


& p = 39


Therefore,





= 13


The length of the perpendicular from the origin to the given plane is





Vector normal to the plane is



Therefore, the unit vector normal to the plane is






Question 7.

Find the Cartesian equation of the plane whose vector equation is


Answer:

Given :


Vector equation of the plane is



To Find : Cartesian equation of the given plane.


Formulae :


1) Dot Product :


If are two vectors




then,



Given the equation of the plane is



Here,




= (x × 3) + (y × 5) + (z × (-9))


= 3x + 5y – 9z


Therefore equation of the plane is


3x + 5y – 9z = 8


This is the Cartesian equation of the given plane.



Question 8.

Find the vector equation of a plane whose Cartesian equation is 5x - 7y + 2z + 4 = 0.


Answer:

Given :


Cartesian equation of the plane is


5x – 7y + 2z + 4 = 0


To Find : Vector equation of the given plane.


Formulae :


1) Dot Product :


If are two vectors




then,



Given the equation of the plane is


5x – 7y + 2z + 4 = 0


⇒ 5x – 7y + 2z = - 4


The term (5x – 7y + 2z) can be written as



But



Therefore the equation of the plane is



or



This is Vector equation of the given plane.



Question 9.

Find a unit vector normal to the plane
x – 2y + 2z = 6.


Answer:

Given :


Equation of plane : x – 2y + 2z = 6


To Find : unit normal vector


Formula :


Unit Vector :


Let be any vector


Then the unit vector of is



Where,


From the given equation of a plane


x – 2y + 2z = 6


direction ratios of vector normal to the plane are (1, -2, 2).


Therefore, the equation of normal vector is



Therefore unit normal vector is given by









Question 10.

Find the direction cosines of the normal to the plane 3x – 6y + 2z = 7.


Answer:

Given :


Equation of plane : 3x – 6y + 2z = 7


To Find : Direction cosines of the normal, i.e.


Formula :


1) Direction cosines :


If a, b & c are direction ratios of the vector, then its direction cosines are given by





For the given equation of a plane


3x – 6y + 2z = 7


Direction ratios of normal vector are (3, -6, 2)






Therefore, direction cosines are







Question 11.

For each of the following planes, find the direction cosines of the normal to the plane and the distance of the plane from the origin:

(i) 2x + 3y - z = 5

(ii) z = 3

(iii) 3y + 5 = 0


Answer:

(i) 2x + 3y – z = 5


Given :


Equation of plane : 2x + 3y – z = 5


To Find :


Direction cosines of the normal i.e.


Distance of the plane from the origin = d


Formulae :


1) Direction cosines :


If a, b & c are direction ratios of the vector then its direction cosines are given by





2) The distance of the plane from the origin :


Distance of the plane from the origin is given by,



For the given equation of plane


2x + 3y – z = 5


Direction ratios of normal vector are (2, 3, -1)


Therefore, equation of normal vector is






Therefore, direction cosines are






Now, the distance of the plane from the origin is




(ii) Given :


Equation of plane : z = 3


To Find :


Direction cosines of the normal, i.e.


The distance of the plane from the origin = d


Formulae :


3) Direction cosines :


If a, b & c are direction ratios of the vector, then its direction cosines are given by





4) The distance of the plane from the origin :


Distance of the plane from the origin is given by,



For the given equation of a plane


z = 3


Direction ratios of normal vector are (0, 0, 1)


Therefore, equation of normal vector is






Therefore, direction cosines are






Now, the distance of the plane from the origin is





(iii) Given :


Equation of plane : 3y + 5 = 0


To Find :


Direction cosines of the normal, i.e.


The distance of the plane from the origin = d


Formulae :


1) Direction cosines :


If a, b & c are direction ratios of the vector, then its direction cosines are given by





2) Distance of the plane from the origin :


Distance of the plane from the origin is given by,



For the given equation of a plane


3y + 5 = 0


⇒-3y = 5


Direction ratios of normal vector are (0, -3, 0)


Therefore, equation of normal vector is






Therefore, direction cosines are






Now, distance of the plane from the origin is





Question 12.

Find the vector and Cartesian equations of the plane passing through the point (2, -1, 1) and perpendicular to the line having direction ratios 4, 2, -3.


Answer:

Given :


A = (2, -1, 1)


Direction ratios of perpendicular vector = (4, 2, -3)


To Find : Equation of a plane


Formulae :


1) Position vectors :


If A is a point having co-ordinates (a1, a2, a3), then its position vector is given by,



2) Dot Product :


If are two vectors




then,



3) Equation of plane :


If a plane is passing through point A, then the equation of a plane is



Where,




For point A = (2, -1, 1), position vector is



Vector perpendicular to the plane with direction ratios (4, 2, -3) is



Now,


= 8 – 2 – 3


= 3


Equation of the plane passing through point A and perpendicular to vector is




As



= 4x + 2y – 3z


Therefore, the equation of the plane is


4x + 2y – 3z = 3


Or


4x + 2y – 3z - 3 = 0



Question 13.

Find the coordinates of the foot of the perpendicular drawn from the origin to the plane

(i) 2x + 3y + 4z -12 = 0

(ii) 5y + 8 = 0


Answer:

(i) 2x + 3y + 4z - 12 = 0


Given :


Equation of plane : 2x + 3y + 4z + 12 = 0


To Find :


coordinates of the foot of the perpendicular


Note :


If two vectors with direction ratios (a1, a2, a3) & (b1, b2, b3) are parallel then



From the given equation of the plane


2x + 3y + 4z – 12 = 0


⇒ 2x + 3y + 4z = 12


Direction ratios of the vector normal to the plane are (2, 3, 4)


Let, P = (x, y, z) be the foot of perpendicular perpendicular drawn from origin to the plane.


Therefore perpendicular drawn is



Let direction ratios of are (x, y, z)


As normal vector and are parallel



⇒x = 2k, y = 3k, z = 4k


As point P lies on the plane, we can write


2(2k) + 3(3k) + 4(4k) = 12


⇒ 4k + 9k + 16k = 12


⇒ 29k = 12



,




Therefore co-ordinates of the foot of perpendicular are


P(x, y, z) =


P =


(ii) Given :


Equation of plane : 5y + 8 = 0


To Find :


coordinates of the foot of the perpendicular


Note :


If two vectors with direction ratios (a1, a2, a3) & (b1, b2, b3) are parallel then



From the given equation of the plane


5y + 8 = 0


⇒ 5y = - 8


Direction ratios of the vector normal to the plane are (0, 5, 0)


Let, P = (x, y, z) be the foot of perpendicular perpendicular drawn from origin to the plane.


Therefore perpendicular drawn is



Let direction ratios of are (x, y, z)


As normal vector and are parallel



⇒x = 0, y = 5k, z = 0


As point P lies on the plane, we can write


5(5k) = -8


⇒ 25k = -8



,




Therefore co-ordinates of the foot of perpendicular are


P(x, y, z) =


P =



Question 14.

Find the length and the foot of perpendicular drawn from the point (2, 3, 7) to the plane 3x – y – z = 7.


Answer:

Given :


Equation of plane : 3x – y – z = 7


A = (2, 3, 7)


To Find :


i) Length of perpendicular = d


ii) coordinates of the foot of the perpendicular


Formulae :


1) Unit Vector :


Let be any vector


Then unit vector of is



Where,


2) Length of perpendicular :


The length of the perpendicular from point A with position vector to the plane is given by,



Note :


If two vectors with direction ratios (a1, a2, a3) & (b1, b2, b3) are parallel then



Given equation of the plane is


3x – y – z = 7 ………..eq(1)


Therefore direction ratios of normal vector of the plane are


(3, -1, -1)


Therefore normal vector of the plane is






From eq(1), p = 7


Given point A = (2, 3, 7)


Position vector of A is



Now,



= (2×3) + (3×(-1)) + (7×(-1))


= 6 – 3 – 7


= -4


Length of the perpendicular from point A to the plane is






Let P be the foot of perpendicular drawn from point A to the given plane,


Let P = (x, y, z)



As normal vector and are parallel



⇒x = 3k+2, y = - k+3, z = -k+7


As point P lies on the plane, we can write


3(3k+2) - (- k+3) - (-k+7) = 7


⇒ 9k + 6 + k – 3 + k – 7 = 7


⇒ 11k = 11



,




Therefore co-ordinates of the foot of perpendicular are


P(x, y, z) =


P = (5, 2, 6)



Question 15.

Find the length and the foot of the perpendicular drawn from the point (1, 1, 2) to the plane


Answer:

Given :


Equation of plane :


A = (1, 1, 2)


To Find :


i) Length of perpendicular = d


ii) coordinates of the foot of the perpendicular


Formulae :


1) Unit Vector :


Let be any vector


Then unit vector of is



Where,


2) Length of perpendicular :


The length of the perpendicular from point A with position vector to the plane is given by,



Note :


If two vectors with direction ratios (a1, a2, a3) & (b1, b2, b3) are parallel then



Given equation of the plane is


………..eq(1)



As


Therefore equation of plane is


2x – 2y + 4z = -5 ……… eq(2)


From eq(1) normal vector of the plane is






From eq(1), p = -5


Given point A = (1, 1, 2)


Position vector of A is



Now,



= (1×2) + (1×(-2)) + (2×4)


= 2 – 2 + 8


= 8


Length of the perpendicular from point A to the plane is








Let P be the foot of perpendicular drawn from point A to the given plane,


Let P = (x, y, z)



As normal vector and are parallel



⇒x = 2k+1, y = -2k+1, z = 4k+2


As point P lies on the plane, we can write


2(2k+1) – 2(-2k+1) + 4(4k+2) = -5


⇒ 4k + 2 + 4k – 2 + 16k + 8 = -5


⇒ 24k = -13



,




Therefore co-ordinates of the foot of perpendicular are


P(x, y, z) =


P ≡



Question 16.

From the point P(1, 2, 4), a perpendicular is drawn on the plane 2x + y - 2z + 3 = 0. Find the equation, the length and the coordinates of the foot of the perpendicular.


Answer:

Given :


Equation of plane : 2x + y – 2z + 3 = 0


P = (1, 2, 4)


To Find :


i) Equation of perpendicular


ii) Length of perpendicular = d


iii) coordinates of the foot of the perpendicular


Formulae :


1) Unit Vector :


Let be any vector


Then unit vector of is



Where,


2) Length of perpendicular :


The length of the perpendicular from point A with position vector to the plane is given by,



Note :


If two vectors with direction ratios (a1, a2, a3) & (b1, b2, b3) are parallel then



Given equation of the plane is


2x + y – 2z + 3 = 0


⇒2x + y – 2z = -3 ………..eq(1)


From eq(1) direction ratios of normal vector of the plane are


(2, 1, -2)


Therefore, equation of normal vector is






= 3


From eq(1), p = -3


Given point P = (1, 2, 4)


Position vector of A is



Here,


Now,



= (1×2) + (2×1) + (4×(-2))


= 2 + 2 - 8


= -4


Length of the perpendicular from point A to the plane is





Let Q be the foot of perpendicular drawn from point P to the given plane,


Let Q = (x, y, z)



As normal vector and are parallel, we can write,



This is the equation of perpendicular.



⇒x = 2k+1, y = k+2, z = -2k+4


As point Q lies on the plane, we can write


2(2k+1) + (k+2) - 2(-2k+4) = -3


⇒ 4k + 2 + k + 2 + 4k - 8 = -3


⇒ 9k = 1



,




Therefore co-ordinates of the foot of perpendicular are


Q(x, y, z) =


Q ≡



Question 17.

Find the coordinates of the foot of the perpendicular and the perpendicular distance from the point P( 3, 2, 1) to the plane 2x – y + z + 1 = 0.

Find also the image of the point P in the plane.


Answer:

Given :


Equation of plane : 2x - y + z + 1 = 0


P = (3, 2, 1)


To Find :


i) Length of perpendicular = d


ii) coordinates of the foot of the perpendicular


iii) Image of point P in the plane.


Formulae :


1) Unit Vector :


Let be any vector


Then unit vector of is



Where,


2) Length of perpendicular :


The length of the perpendicular from point A with position vector to the plane is given by,



Note :


If two vectors with direction ratios (a1, a2, a3) & (b1, b2, b3) are parallel then



Given equation of the plane is


2x - y + z + 1 = 0


⇒2x - y + z = -1 ………..eq(1)


From eq(1) direction ratios of normal vector of the plane are


(2, -1, 1)


Therefore, equation of normal vector is






From eq(1), p = -1


Given point P = (3, 2, 1)


Position vector of A is



Here,


Now,



= (3×2) + (2×(-1)) + (1×1)


= 6 - 2 + 1


= 5


Length of the perpendicular from point A to the plane is






Let Q be the foot of perpendicular drawn from point P to the given plane,


Let Q = (x, y, z)



As normal vector and are parallel, we can write,



⇒x = 2k+3, y = -k+2, z = k+1


As point A lies on the plane, we can write


2(2k+3) - (-k+2) + (k+1) = -1


⇒ 4k + 6 + k – 2 + k + 1 = -1


⇒ 6k = -6



,




Therefore, co-ordinates of the foot of perpendicular are


Q(x, y, z) =


Q ≡


Let, R(a, b, c) be image of point P in the given plane.


Therefore, the power of points P and R in the given plane will be equal and opposite.


2a – b + c + 1 = - (2(3) – 2 + 1 + 1)


⇒2a – b + c + 1 = - 6


⇒2a – b + c = - 7 ………eq(2)


Now,


As are parallel



⇒a = 2k+3, b = -k+2, c = k+1


substituting a, b, c in eq(2)


2(2k+3) - (-k+2) + (k+1) = -7


⇒ 4k + 6 + k – 2 + k + 1 = -7


⇒ 6k = -12



,




Therefore, co-ordinates of the image of P are


R(a, b, c) =


R ≡



Question 18.

Find the coordinates of the image of the point P(1, 3, 4) in the plane 2x - y + z + 3 = 0.


Answer:

Given :


Equation of plane : 2x - y + z + 3 = 0


P = (1, 3, 4)


To Find : Image of point P in the plane.


Note :


If two vectors with direction ratios (a1, a2, a3) & (b1, b2, b3) are parallel then



Given equation of the plane is


2x - y + z + 3 = 0


⇒2x - y + z = -3 ………..eq(1)


From eq(1) direction ratios of normal vector of the plane are


(2, -1, 1)


Therefore, equation of normal vector is



Given point is P = (1, 3, 4)


Let, R(a, b, c) be image of point P in the given plane.


Therefore, the power of points P and R in the given plane will be equal and opposite.


⇒2a – b + c + 3 = - (2(1) – 3 + 4 + 3)


⇒2a – b + c + 3 = - 6


⇒2a – b + c = - 9 ………eq(2)


Now,


As are parallel



⇒a = 2k+1, b = -k+3, c = k+4


substituting a, b, c in eq(2)


2(2k+1) - (-k+3) + (k+4) = -9


⇒ 4k + 2 + k – 3 + k + 4 = -9


⇒ 6k = -12



,




Therefore, co-ordinates of the image of P are


R(a, b, c) =




Question 19.

Find the point where the line meets the plane 2x + 4y – z = 1.


Answer:

Given :


Equation of plane : 2x + 4y - z = 1


Equation of line :



To Find : Point of intersection of line and plane.


Let P(a, b, c) be point of intersection of plane and line.


As point P lies on the line, we can write,



⇒a = 2k+1, b = -3k+2, c = 4k-3 ………(1)


Also point P lies on the plane


2a + 4b – c = 1


⇒2(2k+1) + 4(-3k+2) – (4k-3) = 1 …..from (1)


⇒4k + 2 – 12k + 8 – 4k + 3 = 1


⇒-12k = -12


⇒k = 1


,




Therefore, co-ordinates of point of intersection of given line and plane are


P ≡



Question 20.

Find the coordinates of the point where the line through (3, -4, -5) and (2, -3, 1) crosses the plane 2x + y + z = 7.


Answer:

Given :


Equation of plane : 2x + y + z = 7


Points :


A = (3, -4, -5)


B = (2, -3, 1)


To Find : Point of intersection of line and plane.


Formula :


Equation of line passing through A = (x1, y1, z1) &


B = (x2, y2, z2) is



Equation of line passing through A = (3, -4, -5) & B = (2, -3, 1) is




Let P(a, b, c) be point of intersection of plane and line.


As point P lies on the line, we can write,



⇒a = k+3, b = -k - 4, c = -6k-5 ………(1)


Also point P lies on the plane


2a + b + c = 7


⇒2(k+3) + (-k-4) + (-6k-5) = 7 …..from (1)


⇒2k + 6 – k - 4 – 6k - 5 = 7


⇒-5k = 10


⇒k = -2


,




Therefore, co-ordinates of point of intersection of given line and plane are


P ≡



Question 21.

Find the distance of the point (2, 3, 4) from the plane 3x + 2y + 2z +5 =0, measured parallel to the line


Answer:

Given :


Equation of plane : 3x + 2y + 2z + 5 = 0


Equation of line :



Point : P = (2, 3, 4)


To Find : Distance of point P from the given plane parallel to the given line.


Formula :


1) Equation of line :


Equation of line passing through A = (x1, y1, z1) & having direction ratios (a, b, c) is



2) Distance formula :


The distance between two points A = (a1, a2, a3) & B = (b1, b2, b3) is



For the given line,



Direction ratios are (a, b, c) = (3, 6, 2)


Let Q be the point on the plane such that is parallel to the given line.


Therefore direction ratios of given line and line PQ will be same.


Therefore equation of line PQ with point P = (2, 3, 4) and with direction ratios (3, 6, 2) is



Let co-ordinates of Q be (u, v, w)


As point Q lies on the line PQ, we can write,



⇒u = 3k+2, v = 6k+3, w = 2k+4 ………(1)


Also point Q lies on the plane


3u + 2v + 2w = -5


⇒3(3k+2) + 2(6k+3) + 2(2k+4) = -5 …..from (1)


⇒9k + 6 + 12k + 6 + 4k + 8 = -5


⇒25k = -25


⇒k = -1


,




Therefore, co-ordinates of point Q are


Q =


Now distance between points P and Q by distance formula is






= 7


Therefore distance of point P from the given plane measured parallel to the given line is


d = 7 units



Question 22.

Find the distance of the point (0, -3, 2) from the plane x + 2y -z = 1, measured parallel to the line


Answer:

Given :


Equation of plane : x + 2y - z = 1


Equation of line :



Point : P = (0, -3, 2)


To Find : Distance of point P from the given plane parallel to the given line.


Formula :


1) Equation of line :


Equation of line passing through A = (x1, y1, z1) & having direction ratios (a, b, c) is



2) Distance formula :


The distance between two points A = (a1, a2, a3) & B = (b1, b2, b3) is



For the given line,



Direction ratios are (a, b, c) = (3, 2, 3)


Let Q be the point on the plane such that is parallel to the given line.


Therefore direction ratios of given line and line PQ will be same.


Therefore equation of line PQ with point P = (0, -3, 2) and with direction ratios (3, 2, 3) is



Let co-ordinates of Q be (u, v, w)


As point Q lies on the line PQ, we can write,



⇒u = 3k, v = 2k-3, w = 3k+2 ………(1)


Also point Q lies on the plane


u + 2v - w = 1


⇒(3k) + 2(2k-3) - (3k+2) = 1 …..from (1)


⇒3k + 4k - 6 - 3k - 2 = 1


⇒4k = 9



,




Therefore, co-ordinates of point Q are


Q ≡


Now distance between points P and Q by distance formula is






= 10.55


Therefore distance of point P from the given plane measured parallel to the given line is


d = 10.55 units



Question 23.

Find the equation of the line passing through the point P(4, 6, 2) and the point of intersection of the line and the plane x + y –Z = 8.


Answer:

Given :


Equation of plane : x + y - z = 8


Equation of line :



Point : P = (4, 6, 2)


To Find : Equation of line.


Formula :


Equation of line passing through A = (x1, y1, z1) &


B = (x2, y2, z2) is



let Q (a, b, c) be point of intersection of plane and line.


As point Q lies on the line, we can write,



⇒a =3k+1, b= 2k, c= 7k-1


Also point Q lies on the plane,


a + b – c = 8


⇒(3k+1) + (2k) – (7k-1) = 8


⇒3k + 1 + 2k – 7k + 1 = 8


⇒-2k = 6


⇒k = -3


,




Therefore, co-ordinates of point of intersection of given line and plane are Q = (-8, -6, -22)


Now, equation of line passing through P(4,6,2) and


Q(-8, -6, -22) is





This is the equation of required line



Question 24.

Show that the distance of the point of intersection of the line and the plane x -y + z = 5 from the point (-1, -5 -10) is 13 units.


Answer:

Given :


Equation of plane : x - y + z = 5


Equation of line :



Point : P = (-1, -5, -10)


To Prove : Distance of point P from the given plane parallel to the given line is 13 units.


Formula :


1) Equation of line :


Equation of line passing through A = (x1, y1, z1) & having direction ratios (a, b, c) is



2) Distance formula :


The distance between two points A = (a1, a2, a3) & B = (b1, b2, b3) is



For the given line,



Direction ratios are (a, b, c) = (3, 4, 12)


Let Q be the point on the plane such that is parallel to the given line.


Therefore direction ratios of given line and line PQ will be same.


Therefore equation of line PQ with point P = (-1, -5, -10) and with direction ratios (3, 4, 12) is



Let co-ordinates of Q be (u, v, w)


As point Q lies on the line PQ, we can write,



⇒u = 3k-1, v = 4k-5, w = 12k-10 ………(1)


Also point Q lies on the plane


u - v + w = 5


⇒(3k-1) - (4k-5) + (12k-10) = 5 …..from (1)


⇒3k - 1 - 4k + 5 + 12k - 10 = 5


⇒11k = 11



,




Therefore, co-ordinates of point Q are


Q ≡


Now distance between points P and Q by distance formula is






= 13


Therefore distance of point P from the given plane measured parallel to the given line is


d = 13 units


Hence proved.



Question 25.

Find the distance of the point A(-1, -5, -10) from the point of intersection of the line and the plane

HINT: Convert the equations of the line and the plane to Cartesian form.


Answer:

Given :


Equation of plane :


Equation of line :



Point : P = (-1, -5, -10)


To Find : Distance of point P from the given plane parallel to the given line.


Formula :


1) Equation of line :


Equation of line passing through A = (x1, y1, z1) & having direction ratios (a, b, c) is



2) Distance formula :


The distance between two points A = (a1, a2, a3) & B = (b1, b2, b3) is



for the given plane,



Here,



⇒x – y + z = 5 ………eq(1)


For the given line,



Here,




Comparing coefficients of



………eq(2)


Direction ratios for above line are (a, b, c) = (3, 4, 2)


Let Q be the point on the plane such that is parallel to the given line.


Therefore direction ratios of given line and line PQ will be same.


Therefore equation of line PQ with point P = (-1, -5, -10) and with direction ratios (3, 4, 2) is



Let co-ordinates of Q be (u, v, w)


As point Q lies on the line PQ, we can write,



⇒u = 3k-1, v = 4k-5, w = 2k-10 ………(3)


Also point Q lies on the given plane


Therefore from eq(1), we can write,


u - v + w = 5


⇒(3k-1) - (4k-5) + (2k-10) = 5 …..from (3)


⇒3k - 1 - 4k + 5 + 2k - 10 = 5


⇒k = 11



,




Therefore, co-ordinates of point Q are


Q ≡


Now the distance between points P and Q by distance formula is






= 59.24


Therefore distance of point P from the given plane measured parallel to the given line is


d = 59.24 units



Question 26.

Prove that the normals to the planes 4x + 11y + 2z + 3 = 0 and 3x - 2y + 5z = 8 are perpendicular to each other.


Answer:

Given :


Equations of plane are :


4x + 11y + 2z + 3 = 0


3x – 2y + 5z = 8


To Prove : are perpendicular.


Formula :


1) Dot Product :


If are two vectors




then,



Note :


Direction ratios of the plane given by


ax + by + cz = d


are (a, b, c).


For plane


4x + 11y + 2z + 3 = 0


direction ratios of normal vector are (4, 11, 2)


therefore, equation of normal vector is



And for plane


3x – 2y + 5z = 8


direction ratios of the normal vector are (3, -2, 5)


therefore, the equation of normal vector is



Now,




= 12 – 22 + 10


= 0



Therefore, normals to the given planes are perpendicular.



Question 27.

Show that the line is parallel to the plane


Answer:

Given :


Equation of plane : :


Equation of a line :



To Prove : Given line is parallel to the given plane.


Comparing given plane i.e.



with , we get,



This is the vector perpendicular to the given plane.


Now, comparing the given the equation of line i.e.



with , we get,



Now,




= 1 – 5 + 4


= 0



Therefore, a vector normal to the plane is perpendicular to the vector parallel to the line.


Hence, the given line is parallel to the given plane.



Question 28.

Find the equation of a plane which is at a distance of 3√3 units from the origin and the normal to which is equally inclined to the coordinate axes.


Answer:

Given :




To Find : Equation of plane


Formulae :


1) Distance of plane from the origin :


If is the vector normal to the plane, then distance of the plane from the origin is



Where,


2)


Where


3) Equation of plane :


If is the vector normal to the plane, then equation of the plane is



As



⇒ l = m = n





Therefore equation of normal vector of the plane having direction cosines l, m, n is






= √1


= 1


Now,


distance of the plane from the origin is





Therefore equation of required plane is







This is the required equation of the plane.



Question 29.

A vector of magnitude 8 units is inclined to the x-axis at 45o, y-axis at 60o and an acute angle with the z-axis, if a plane passes through a point (√2, -1, 1) and is normal to find its equation in vector form.


Answer:

Given :





P = (√2, -1, 1)


To Find : Equation of plane


Formulae :


1)


Where


2) Equation of plane :


If is the vector normal to the plane, then equation of the plane is



As


and



But,







Therefore direction cosines of the normal vector of the plane are (l, m, n)


Hence direction ratios are (kl, km, kn)


Therefore the equation of normal vector is










Now, equation of the plane is



………eq(1)


But



⇒4√2x + 4y + 4z = p


As point P (√2, -1, 1) lies on the plane by substituting it in above equation,


4√2(√2) + 4(-1) + 4(1) = p


⇒8 – 4 + 4 = p


⇒P = 8


From eq(1)



Dividing throughout by 4



This is the equation of required plane.



Question 30.

Find the vector equation of a line passing through the point and perpendicular to the plane

Also, find the point of intersection of this line and the plane.


Answer:

Given :



Equation of plane :


To Find :


Equation of line


Point of intersection


Formula :


Equation of line passing through point A with position vector and parallel to vector is



Where,


From the given equation of the plane


………eq(1)


The normal vector of the plane is



As the given line is perpendicular to the plane therefore will be parallel to the line.



Now, the equation of the line passing through and parallel to is




………eq(2)


This is the required equation line.


Substituting in eq(1)



⇒6x – 3y + 5z = -2 ………eq(3)


Also substituting in eq(2)





Comparing coefficients of



………eq(4)


Let Q(a, b, c) be the point of intersection of given line and plane


As point Q lies on the given line.


Therefore from eq(4)



⇒a = 6k+2, b = -3k-3, c = 5k-5


Also point Q lies on the plane.


Therefore from eq(3)


6a – 3b + 5c = -2


⇒6(6k+2) – 3(-3k-3) + 5(5k-5) = -2


⇒36k + 12 + 9k + 9 + 25k – 25 = -2


⇒70k = 2






Therefore co-ordinates of the point of intersection of line and plane are


Q ≡




Exercise 28c
Question 1.

Find the distance of the point from the plane


Answer:

Formula :


where () is point from which distance is to be calculated


Therefore ,


Plane can be written in cartesian form as


3x - 4y + 12z = 9


3x - 4y + 12z - 9 = 0


Point = (2i - j - 4k)


Which can be also written as


Point = ( 2 , - 1 , - 4 )


Distance






Question 2.

Find the distance of the point from the plane


Answer:

Formula :


where () is point from which distance is to be calculated


Therefore ,


Plane can be written in cartesian form as


x + y + z + 17 = 0


Point = (i + 2j + 5k)


Which can be also written as


Point = ( 1 , 2 , 5 )


Distance






Question 3.

Find the distance of the point (3, 4, 5) from the plane


Answer:

Formula :


where () is point from which distance is to be calculated


Therefore ,


Plane can be written in cartesian form as


2x - 5y + 3z = 13


2x - 5y + 3z - 13 = 0


Point = ( 3 , 4 , 5 )


Distance






Question 4.

Find the distance of the point (1, 1, 2) from the plane


Answer:

Formula :


where () is point from which distance is to be calculated


Therefore ,


Plane can be written in cartesian form as


2x - 2y + 4z + 5 = 0


Point = ( 1 , 1 , 2 )


Distance






Question 5.

Find the distance of the point (2, 1, 0) from the plane 2x + y + 2z + 5 = 0.


Answer:

Formula :


where () is point from which distance is to be calculated


Therefore ,


2x + y + 2z + 5 = 0


Point = ( 2 , 1 , 0 )


Distance






Question 6.

Find the distance of the point (2, 1, - 1) from the plane x – 2y + 4z = 9.


Answer:

Formula :


where () is point from which distance is to be calculated


Therefore ,


x - 2y + 4z = 9


x - 2y + 4z – 9 = 0


Point = ( 2 , 1 , - 1 )


Distance






Question 7.

Show that the point (1, 2, 1) is equidistant from the planes and


Answer:

Formula :


where () is point from which distance is to be calculated


Therefore ,


First Plane can be written in cartesian form as


x + 2y - 2z = 5


x + 2y - 2z – 5 = 0


Point = ( 1 , 2 , 1 )


Distance for first plane





Second Plane can be written in cartesian form as


2x - 2y + z + 3 = 0


Point = ( 1 , 2 , 1 )


Distance for second plane





Hence proved.



Question 8.

Show that the points ( - 3, 0, 1) and (1, 1, 1) are equidistant from the plane 3x + 4y – 12z + 13 = 0.


Answer:

Formula :


where () is point from which distance is to be calculated


Therefore ,


Plane = 3x + 4y - 12z + 13 = 0


First Point = ( - 3 , 0 , 1 )


Distance for first point





Plane = 3x + 4y - 12z + 13 = 0


Second Point = ( 1 , 1 , 1 )


Distance for first point





Hence proved.



Question 9.

Find the distance between the parallel planes 2x + 3y + 4 = 4 and 4x + 6y + 8z = 12.


Answer:

Formula : The distance between two parallel planes, say


Plane 1:ax + by + cz + d1 = 0 &


Plane 2:ax + by + cz + d2 = 0 is given by the formula



where () are constants of the planes


Therefore ,


First Plane 2x + 3y + 4 = 4


2x + 3y + 4 – 4 = 0 …… (1)


Second plane 4x + 6y + 8z = 12


4x + 6y + 8z - 12 = 0


2(2x + 3y + 4z – 6) = 0


2x + 3y + 4z – 6 = 0 …… (2)


Using equation (1) and (2)


Distance between both planes






Question 10.

Find the distance between the parallel planes x + 2y - 2z + 4 = 0 and x + 2y – 2z – 8 = 0.


Answer:

Formula : The distance between two parallel planes, say


Plane 1:ax + by + cz + d1 = 0 &


Palne 2:ax + by + cz + d2 = 0 is given by the formula



where () are costants of the planes


Therefore ,


First Plane x + 2y - 2z + 4 = 0 …… (1)


Second plane x + 2y – 2z – 8 = 0 …… (2)


Using equation (1) and (2)


Distance between both planes






Question 11.

Find the equation of the planes parallel to the plane x – 2y + 2z – 3 = 0, each one of which is at a unit distance from the point (1, 1, 1).


Answer:

Formula : Plane = r . (n) = d


Where r = any random point


n = normal vector of plane


d = distance of plane from origin


If two planes are parallel , then their normal vectors are same


Therefore ,


Parallel Plane x – 2y + 2z – 3 = 0


Normal vector = (i - 2j + 2k)


∴ Normal vector of required plane = (i - 2j + 2k)


Equation of required planes r . (i - 2j + 2k) = d


In cartesian form x – 2y + 2y = d


It should be at unit distance from point (1,1,1)


Distance






For + sign = > 3 = 1 - d = > d = - 2


For - sign = > 3 = - 1 + d = > d = 4


Therefore equations of planes are : -


For d = - 2 For d = 4


x – 2y + 2y = d x – 2y + 2y = d


x – 2y + 2y = - 2 x – 2y + 2y = 4


x – 2y + 2y + 2 = 0 x – 2y + 2y – 4 = 0


Required planes = x – 2y + 2y + 2 = 0


x – 2y + 2y – 4 = 0



Question 12.

Find the equation of the plane parallel to the plane 2x – 3y + 5z + 7 = 0 and passing through the point (3, 4, - 1). Also, find the distance between the two planes.


Answer:

Formula : Plane = r . (n) = d


Where r = any random point


n = normal vector of plane


d = distance of plane from origin


The distance between two parallel planes, say


Plane 1:ax + by + cz + d1 = 0 &


Palne 2:ax + by + cz + d2 = 0 is given by the formula



If two planes are parallel , then their normal vectors are same


Therefore ,


Parallel Plane 2x – 3y + 5z + 7 = 0


Normal vector = (2i - 3j + 5k)


∴ Normal vector of required plane = (2i - 3j + 5k)


Equation of required plane r . (2i - 3j + 5k) = d


In cartesian form 2x – 3y + 5y = d


Plane passes through point (3,4, - 1) therefore it will satisfy it.


2(3) – 3(4) + 5( - 1) = d


6 – 12 - 5 = d


d = - 11


Equation of required plane 2x – 3y + 5z = - 11


2x – 3y + 5z + 11 = 0


Therefore ,


First Plane 2x – 3y + 5z + 7 = 0 …… (1)


Second plane 2x – 3y + 5z + 11 = 0 …… (2)


Using equation (1) and (2)


Distance between both planes






Question 13.

Find the equation of the plane mid - parallel to the planes 2x – 3y + 6z + 21 = 0 and 2x – 3y + 6z – 14 = 0


Answer:

Formula : The equation of mid parallel plane is , say


Plane 1:ax + by + cz + d1 = 0 &


Plane 2:ax + by + cz + d2 = 0 is given by the formula



where () are constants of the planes


Therefore ,


First Plane 2x – 3y + 6z + 21 = 0 …… (1)


Second plane 2x – 3y + 6z – 14 = 0 …… (2)


Using equation (1) and (2)


Mid parallel plane


4x – 6y + 12z + 7 = 0




Exercise 28d
Question 1.

Show that the planes 2x – y + 6z = 5 and 5x – 2.5y + 15z = 12 are parallel.


Answer:

Formula : Plane = r . (n) = d


Where r = any random point


n = normal vector of plane


d = distance of plane from origin


If two planes are parallel , then their normal vectors are either same or proportional to each other


Therefore ,


Plane 1 : - 2x – y + 6z = 5


Normal vector (Plane 1) = (2i – j + 6k) …..(1)


Plane 2 : - 5x – 2.5y + 15z = 12


Normal vector (Plane 2) = (5i – 2.5j + 15k) …..(2)


Multiply equation(1) by 5 and equation(2) by 2


Normal vector (Plane 1) = 5(2i – j + 6k)


= 10i – 5j + 30k


Normal vector (Plane 2) = 2(5i – 2.5j + 15k)


= 10i – 5j + 30k


Since, both normal vectors are same .Therefore both planes are parallel



Question 2.

Find the vector equation of the plane through the point and parallel to the plane


Answer:

Formula : Plane = r . (n) = d


Where r = any random point


n = normal vector of plane


d = distance of plane from origin


If two planes are parallel , then their normal vectors are same.


Therefore ,


Parallel Plane r . (2i – 3j + 5k) + 5 = 0


Normal vector = (2i - 3j + 5k)


∴ Normal vector of required plane = (2i - 3j + 5k)


Equation of required plane r . (2i - 3j + 5k) = d


In cartesian form 2x – 3y + 5z = d


Plane passes through point (3,4, - 1) therefore it will satisfy it.


2(3) – 3(4) + 5( - 1) = d


6 – 12 - 5 = d


d = - 11


Equation of required plane r . (2i - 3j + 5k) = - 11


r . (2i - 3j + 5k) + 11 = 0



Question 3.

Find the vector equation of the plane passing through the point (a, b, b) and parallel to the plane

There is a error in question …… the point should be (a,b,c) instead of (a,b,b) to get the required answer.


Answer:

Formula : Plane = r . (n) = d


Where r = any random point


n = normal vector of plane


d = distance of plane from origin


If two planes are parallel , then their normal vectors are same.


Therefore ,


Parallel Plane r . (i + j + k) = 2


Normal vector = (i + j + k)


∴ Normal vector of required plane = (i + j + k)


Equation of required plane r . (i + j + k) = d


In cartesian form x + y + z = d


Plane passes through point (a,b,c) therefore it will satisfy it.


(a) + (b) + (c) = d


d = a + b + c


Equation of required plane r . (i + j + k) = a + b + c



Question 4.

Find the vector equation of the plane passing through the point (1, 1, 1) and parallel to the plane


Answer:

Formula : Plane = r . (n) = d


Where r = any random point


n = normal vector of plane


d = distance of plane from origin


If two planes are parallel , then their normal vectors are same.


Therefore ,


Parallel Plane r . (2i - j + 2k) = 5


Normal vector = (2i - j + 2k)


∴ Normal vector of required plane = (2i - j + 2k)


Equation of required plane r . (2i - j + 2k) = d


In cartesian form 2x - y + 2z = d


Plane passes through point (1,1,1) therefore it will satisfy it.


2(1) - (1) + 2(1) = d


d = 2 – 1 + 2 = 3


Equation of required plane r . (2i - j + 2k) = 3



Question 5.

Find the equation of the plane passing through the point (1, 4, - 2) and parallel to the plane 2x – y + 3z + 7 = 0.


Answer:

Formula : Plane = r . (n) = d


Where r = any random point


n = normal vector of plane


d = distance of plane from origin


If two planes are parallel , then their normal vectors are same.


Therefore ,


Parallel Plane 2x – y + 3z + 7 = 0


Normal vector = (2i - j + 3k)


∴ Normal vector of required plane = (2i - j + 3k)


Equation of required plane r . (2i - j + 3k) = d


In cartesian form 2x - y + 3z = d


Plane passes through point (1,4, - 2) therefore it will satisfy it.


2(1) - (4) + 3( - 2) = d


d = 2 – 4 – 6 = - 8


Equation of required plane 2x - y + 3z = - 8


2x - y + 3z + 8 = 0



Question 6.

Find the equations of the plane passing through the origin and parallel to the plane 2x – 3y + 7z + 13 = 0.


Answer:

Formula : Plane = r . (n) = d


Where r = any random point


n = normal vector of plane


d = distance of plane from origin


If two planes are parallel , then their normal vectors are same.


Therefore ,


Parallel Plane 2x – 3y + 7z + 13 = 0


Normal vector = (2i - 3j + 7k)


∴ Normal vector of required plane = (2i - 3j + 7k)


Equation of required plane r . (2i - 3j + 7k) = d


In cartesian form 2x - 3y + 7z = d


Plane passes through point (0,0,0) therefore it will satisfy it.


2(0) - (0) + 3(0) = d


d = 0


Equation of required plane 2x - 3y + 7z = 0



Question 7.

Find the equations of the plane passing through the point ( - 1, 0, 7) and parallel to the plane 3x – 5y + 4z = 11.


Answer:

Formula : Plane = r . (n) = d


Where r = any random point


n = normal vector of plane


d = distance of plane from origin


If two planes are parallel , then their normal vectors are same.


Therefore ,


Parallel Plane 3x – 5y + 4z = 11


Normal vector = (3i - 5j + 4k)


∴ Normal vector of required plane = (3i - 5j + 4k)


Equation of required plane r . (3i - 5j + 4k) = d


In cartesian form 3x - 5y + 4z = d


Plane passes through point ( - 1,0,7) therefore it will satisfy it.


3( - 1) - 5(0) + 4(7) = d


d = - 3 + 28 = 25


Equation of required plane 3x - 5y + 4z = 25



Question 8.

Find the equations of planes parallel to the plane x – 2y + 2z = 3 which are at a unit distance from the point (1, 2, 3).


Answer:

Formula : Plane = r . (n) = d


Where r = any random point


n = normal vector of plane


d = distance of plane from origin


If two planes are parallel , then their normal vectors are same


Therefore ,


Parallel Plane x – 2y + 2z – 3 = 0


Normal vector = (i - 2j + 2k)


∴ Normal vector of required plane = (i - 2j + 2k)


Equation of required planes r . (i - 2j + 2k) = d


In cartesian form x – 2y + 2y = d


It should be at unit distance from point (1,2,3)


Distance






For + sign = > 3 = 3 - d = > d = 0


For - sign = > 3 = - 3 + d = > d = 6


Therefore equations of planes are : -


For d = 0 For d = 6


x – 2y + 2y = d x – 2y + 2y = d


x – 2y + 2y = 0 x – 2y + 2y = 6


Required planes = x – 2y + 2y = 0


x – 2y + 2y – 6 = 0



Question 9.

Find the distance between the planes x + 2y + 3z + 7 = 0 and 2x + 4y + 6z + 7 = 0.


Answer:

Formula : The distance between two parallel planes, say


Plane 1:ax + by + cz + d1 = 0 &


Plane 2:ax + by + cz + d2 = 0 is given by the formula



where () are constants of the planes


Therefore ,


First Plane x + 2y + 3z + 7 = 0


2(x + 2y + 3z + 7) = 0


2x + 4y + 6z + 14 = 0 …… (1)


Second plane 2x + 4y + 6z + 7 = 0 …… (2)


Using equation (1) and (2)


Distance between both planes







Exercise 28e
Question 1.

Find the equation of the plane through the line of intersection of the planes x + y + z = 6 and 2x + 2y + 4z + 5 = 0, and passing through the point (1, 1, 1).


Answer:

Equation of plane through the line of intersection of planes in Cartesian form is

(1)


For the standard equation of planes,



So, putting in equation (1), we have


x + y + z-6 + λ(2x + 2y + 4z + 5)=0


(1 + 2λ)x + (1 + 2λ)y + (1 + 4λ)z-6 + 5λ=0 (2)


Now plane passes through (1,1,1) then it must satisfy the plane equation,


(1 + 2λ).1 + (1 + 2λ).1 + (1 + 4λ).1-6 + 5λ=0


1 + 2λ + 1 + 2λ + 1 + 4λ-6 + 5λ=0


3 + 8λ-6 + 5λ=0


13λ=3



Putting in equation (2)




19x + 19y + 25z-63=0


So, the required equation of plane is 19x + 19y + 25z=63.



Question 2.

Find the equation of the plane through the line of intersection of the planes x - 3y + z + 6 = 0 and x + 2y + 3z + 5 = 0, and passing through the origin.


Answer:

Equation of plane through the line of intersection of planes in Cartesian form is

(1)


For the standard equation of planes,



So, putting in equation (1), we have


x-3y + z + 6 + λ(x + 2y + 3z + 5)=0


(1 + λ)x + (-3 + 2λ)y + (1 + 3λ)z + 6 + 5λ=0 (2)


Now plane passes through (0,0,0) then it must satisfy the plane equation,


(1 + λ).0 + (-3 + 2λ).0 + (1 + 3λ).0 + 6 + 5λ=0


5λ=-6



Putting in equation (2)




-x-27y-13z=0


x + 27y + 13z=0


So, required equation of plane is x + 27y + 13z=0.



Question 3.

Find the equation of the plane passing through the intersection of the planes 2x + 3y – z + 1 = 0 and x + y – 2z + 3 = 0, and perpendicular to the plane 3x - y -2z -4 = 0.


Answer:

Equation of plane through the line of intersection of planes in Cartesian form is

(1)


For the standard equation of planes,



So, putting in equation (1), we have


2x + 3y–z + 1 + λ(x + y–2z + 3)=0


(2 + λ)x + (3 + λ)y + (-1-2λ)z + 1 + 3λ=0 (2)


Now as the plane 3x-y-2z-4=0 is perpendicular the given plane,


For θ=90°, cos90°=0


(3)


On comparing with standard equations in Cartesian form,



Putting values in equation (3), we have


(2 + λ).3 + (3 + λ).(-1) + (-1-2λ).(-2)=0


6 + 3λ-3-λ + 2 + 4λ=0


5 + 6λ=0



Putting in equation(2)




7x + 13y + 4z-9=0


7x + 13y + 4z=9


So, required equation of plane is 7x + 13y + 4z=9.



Question 4.

Find the equation of the plane passing through the line of intersection of the planes 2x - y = 0 and 3z - y = 0, and perpendicular to the plane 4x + 5y - 3z = 9.


Answer:

Equation of plane through the line of intersection of planes in Cartesian form is

(1)


For the standard equation of planes,



So, putting in equation (1), we have


2x-y + λ(3z-y)=0


2x + (-1-λ)y + 3λz=0 (2)


Now as the plane is perpendicular the given plane,


For θ=90°, cos90°=0


(3)


On comparing with standard equations in Cartesian form,



Putting values in equation(3),


2.4 + (-1-λ).5 + 3λ.-3=0


8-5-5λ-9λ=0


-14λ=-3



Putting in equation(2)




28x-17y + 9z=0


So, required equation of plane is 28x-17y + 9z=0.



Question 5.

Find the equation of the plane passing through the intersection of the planes x - 2y + z = 1 and 2x + y + z = 8, and parallel to the line with direction ratios 1, 2, 1. Also, find the perpendicular distance of (1, 1, 1) from the plane.


Answer:

Equation of plane through the line of intersection of planes in Cartesian form is

(1)


For the standard equation of planes,



So, putting in equation (1), we have


x-2y + z-1 + λ(2x + y + z-8)=0


(1 + 2λ)x + (-2 + λ)y + (1 + λ)z-1-8λ=0 (2)


For plane the normal is perpendicular to line given parallel to this i.e.



Where A1, B1, C1 are direction ratios of plane and A2, B2, C2 are of line.


(1 + 2λ).1 + (-2 + λ).2 + (1 + λ).1=0


1 + 2λ-4 + 2λ + 1 + λ=0


-2 + 5λ=0



Putting the value of λ in equation (2)




9x-8y + 7z-21=0


9x-8y + 7z=21


For the equation of plane Ax + By + Cz=D and point (x1,y1,z1), a distance of a point from a plane can be calculated as




So, the required equation of the plane is 9x-8y + 7z=21, and distance of the plane from (1,1,1) is




Question 6.

Find the equation of the plane passing through the line of intersection of the planes x + 2y + 3z – 5 = 0 and 3x - 2y –z + 1 = 0 and cutting off equal intercepts on the x-axis and z-axis.


Answer:

Equation of plane through the line of intersection of planes in Cartesian form is

(1)


For the standard equation of planes in Cartesian form



So, putting in equation 1 we have


x + 2y + 3 z-5 + λ(3x -2y -z + 1)=0


(1 + 3λ)x + (2-2λ)y + (3-λ)z-5 + λ=0


Now equation of plane in intercept form



As given equal intercept means a=c


First, we transform equation of a plane in intercept form




On comparing with the standard equation of a plane in intercept form



Now as a=b=c




Putting in equation (2), we have




5x + 2y + 5z-9=0


5x + 2y + 5z=9


So, required equation of plane is 5x + 2y + 5z=9.



Question 7.

Find the equation of the plane through the intersection of the planes 3x – 4y + 5z =10 and 2x + 2y - 3z = 4 and parallel to the line x = 2y = 3z.


Answer:

Equation of plane through the line of intersection of planes in Cartesian form is

(1)


For the standard equation of planes in Cartesian form



So, putting in equation (1), we have


3x – 4y + 5z -10 + λ(2x + 2y - 3z - 4)=0


(3 + 2λ)x + (-4 + 2λ)y + (5-3λ)z-10-4λ=0


Given line is parallel to plane then the normal of plane is perpendicular to line,



Where A1, B1, C1 are direction ratios of plane and A2, B2, C2 are of line.


(3 + 2λ).6 + (-4 + 2λ).3 + (5-3λ).2=0


18 + 12λ-12 + 6λ + 10-6λ=0


16 + 12λ=0



Putting the value of λ in equation (2)




x-20y + 27z-14=0


So, required equation of plane is x-20y + 27z-14=0.



Question 8.

Find the vector equation of the plane through the intersection of the planes and and passing through the point (2, 1, -1).


Answer:

Equation of plane through the line of intersection of two planes in vector form is

(1)


Where the standard equation of planes are



Putting values in equation(1)



(2)


Now as the plane passes through (2,1,-1)



Putting in equation (2)



2.1 + 1.(3 + λ) + (-1)(-1 + 2λ)=0


2 + 3 + λ + 1-2λ=0


λ=6


Putting the value of λ in equation (2)




.



Question 9.

Find the vector equation of the plane through the point (1, 1, 1), and passing through the intersection of the planes and


Answer:

Equation of plane through the line of intersection of two planes in vector form is

(1)


Where the standard equation of planes are



Putting values in equation(1)



(2)


Now as the plane passes through (1,1,1)



Putting in equation (2)



1.(1 + 2λ) + 1.(-1 + λ) + 1.(3-λ)=-1 + 5λ


1 + 2λ-1 + λ + 3- λ + 1-5λ=0


-3λ + 4=0



Putting the value of λ in equation (2)







Question 10.

Find the vector equation of the plane passing through the intersection of the planes and and passing through the point (-2, 1, 3).


Answer:

Equation of plane through the line of intersection of two planes in vector form is

(1)


Where the standard equation of planes are



Putting values in equation(1)



(2)


Now as the plane passes through (-2,1,3)



Putting in equation (2)



-2.(2 + 3λ) + 1.(-7-5λ) + 3.(4 + 4λ)=3-11λ


-4-6λ-7-5λ + 12 + 12λ-3 + 11λ=0


-14 + 12 + 12λ=0



Putting the value of λ in equation (2)







Question 11.

Find the equation of the plane through the line of intersection of the planes and and perpendicular to the plane


Answer:

Equation of plane through the line of intersection of two planes in vector form is

(1)


Where the standard equation of planes are



Putting values in equation (1), we have



(2)


Given a plane perpendicular to this plane, So if n1 and n2 are normal


vectors of planes



(


2.(2 + λ) + (-1).(-3-λ) + 1.4=0


4 + 2λ + 3 + λ + 4=0


11 + 3λ=0



Putting the value of λ in equation (2)







Question 12.

Find the Cartesian and vector equations of the planes through the line of intersection of the planes and which are at a unit distance from the origin.


Answer:

Equation of plane through the line of intersection of two planes in vector form is

(1)


Where the standard equation of planes are



Putting values in equation (1)



(2)


For the equation of plane Ax + By + Cz=D and point (x1,y1,z1), a distance of a point from a plane can be calculated as










Putting value of λ in equation (2)


λ=1




λ=-1




For equations in Cartesian form put



For λ=1



x.2 + y.1 + z.(-2)-3=0


2x + y-2z-3=0


For λ=-1



x.1 + y.2 + z.(-2) + 3=0


x + 2y-2z + 3=0





In Cartesian form are 2x + y-2z-3=0 & x + 2y-2z + 3=0




Exercise 28f
Question 1.

Find the acute angle between the following planes :

(i) and

(ii) and

(iii) and

(iv) and


Answer:

To find the angle between two planes, we simply find the angle between the normal vectors of planes. So if n1 and n2 are normal vectors and θ is the angle between both then,


(i)On comparing with the standard equation of planes in vector form




Then





(ii) On comparing with the standard equation of planes in vector form




Then





(iii) On comparing with the standard equation of planes in vector form




Then





(iv)On comparing with the standard equation of planes in vector for




Then







Question 2.

Show that the following planes are at right angles:

(i) and

(ii) and


Answer:

To show the right angle between two planes, we simply find the angle between the normal vectors of planes. So if n1 and n2 are normal vectors and θ is the angle between both then

.for right angle θ=90°


Cos90°=0


(1)


(i)On comparing with standard equation





Hence proved planes at right angles.


(ii) On comparing with the standard equation of a plane





Hence proved planes at right angles.



Question 3.

Find the value of λ for which the given planes are perpendicular to each other:

(i) and

(ii) and


Answer:

For planes perpendicular Cos90°=0

(1)


(i)On comparing with the standard equation of a plane




2.3 + (-1).2 + (-λ).2=0


6-2-2λ=0


2λ=4


λ=2


(ii) On comparing with the standard equation of a plane




λ.1 + 2.2 + 3.(-7)=0 λ + 4-21=0 λ=17



Question 4.

Find the acute angle between the following planes:

(i) 2X – y + z = 5 and x + y + 2z = 7

(ii) x + 2y + 2z = 3 and 2x - 3y + 6z = 8

(iii) x + y - z = 4 and x + 2y + z = 9

(iv) x + y - 2z = 6 and 2x - 2y + z = 11


Answer:

To find angle in Cartesian form, for standard equation of planes



(i)On comparing with the standard equation of planes






(ii)On comparing with the standard equation of planes






(iii) On comparing with standard equation of planes






(iv)On comparing with the standard equation of planes







Question 5.

Show that each of the following pairs of planes are at right angles:

(i) 3x + 4y - 5z = 7 and 2x + 6y + 6z + 7 = 0

(ii) x - 2y + 4z = 10 and 18x + 17y + 4z = 49


Answer:

To find angle in Cartesian form, for standard equation of planes


For θ=90°, cos90°=0



(i)On comparing with the standard equation of a plane




=0=RHS


Hence proved that the angle between planes is 90°.


(ii) On comparing with the standard equation of a plane




=0=RHS


Hence proved that angle between planes is 90°.



Question 6.

Prove that the plane 2x + 2y + 4z = 9 is perpendicular to each of the planes x + 2y + 2z – 7 = 0 and 5x + 6y + 7z = 23.


Answer:

To show that planes are perpendicular


Where A1, B1, C1 are direction ratios of plane and A2, B2, C2 are of other


plane.


2.1 + 2.2 + 4.2=2 + 4 + 8=14≠0


Hence, planes are not perpendicular.


Similarly for the other plane


2.5 + 2.6 + 2.7=10 + 12 + 14=36≠0


Hence, planes are not perpendicular.



Question 7.

Show that the planes 2x – 2y + 4z + 5 = 0 and 3x – 3y + 6z - 1 = 0 are parallel.


Answer:

To show that planes are parallel


On comparing with the standard equation of a plane




So,



Hence proved that planes are parallel.



Question 8.

Find the value of λ for which the planes x – 4y + λz + 3 = 0 and 2x + 2y + 3z = 5 are perpendicular to each other.


Answer:

To find an angle in Cartesian form, for the standard equation of planes



For θ=90°, cos90°=0



On comparing with the standard equation of the plane,




2 + (-8) + 3λ=0


-6 + 3λ=0


λ=2



Question 9.

Write the equation of the plane passing through the origin and parallel to the plane 5x - 3y + 7z + 11 = 0.


Answer:

Let the equation of plane be


Direction ratios of parallel planes are related to each other as



Putting the values from the equation of a given parallel plane,




Putting in equation plane



As the plane is passing through (0,0,0), it must satisfy the plane,




5kx-3ky + 7kz=0


5x-3y + 7z=0


So, required equation of plane is 5x-3y + 7z=0.



Question 10.

Find the equation of the plane passing through the point (a, b, c) and parallel to the plane


Answer:

Let the equation of a plane

(1)


Direction ratios of parallel planes are related to each other as



Putting the values from the equation of a given parallel plane,




Putting values in equation (1), we have


(2)


A plane passes through (a,b,c) then it must satisfy the equation of a plane




λ(a.1 + b.1 + c.1)=d


λ(a + b + c)=d


Putting value in equation (2)






Question 11.

Find the equation of the plane passing through the point (1, -2, 7) and parallel to the plane 5x + 4y - 11z = 6.


Answer:

Let the equation of plane be


Direction ratios of parallel planes are related to each other as



Putting the values from the equation of a given parallel plane,




Putting in the equation of a plane



As the plane is passing through (1,-2,7), it must satisfy the plane,


(1)




Putting value in equation (1), we have



5x + 4y-11z + 80=0


So, the required equation of the plane is 5x + 4y-11z + 80=0.



Question 12.

Find the equation of the plane passing through the point A(-1, -1, 2) and perpendicular to each of the planes 3x + 2y - 3z = 1 and 5x – 4y + z = 5.


Answer:

Applying the condition of perpendicularity between planes


Where A, B, C are direction ratios of plane and A1, B1, C1 are of another plane.


(1)


(2)


And plane passes through (-1,-1,2),


A(x + 1) + B(y + 1) + C(z-2)=0 (3)


On solving equation (1) and (2)



Putting values in equation (3)



B(5x + 5 + 9y + 9 + 11z-22)=0


5x + 9y + 11z-8=0


So, required equation of plane is 5x + 9y + 11z=8.



Question 13.

Find the equation of the plane passing through the origin and perpendicular to each of the planes x + 2y - z = 1 and 3x - 4y + z = 5.


Answer:

Applying condition of perpendicularity between planes,


Where A, B, C are direction ratios of plane and A1, B1, C1 are of other


plane.



(1)



(2)


And plane passes through (0, 0, 0),


A(x-0) + B(y-0) + C(z-0)=0


Ax + By + Cz=0 (3)


On solving equation (1) and (2)



Putting values in equation(3)



B(x + 2y + 5z)=0


x + 2y + 5z=0


So, required equation of plane is x + 2y + 5z=0.



Question 14.

Find the equation of the plane that contains the point A(1, -1, 2) and is perpendicular to both the planes 3x + 3y – 2z = 5 and x + 2y - 3z = 8. Hence, find the distance of the point P(-2, 5, 5) from the plane obtained above.


Answer:

Applying condition of perpendicularity between planes,


Where A, B, C are direction ratios of plane and A1, B1, C1 are of other


plane.



(1)



(2)


And plane contains the point (1,-1,2),


A(x-1) + B(y + 1) + C(z-2)=0 (3)


On solving equation (1) and (2)



Putting values in equation (3)




-5x + 5 + 7y + 7 + 3z-6=0


-5x + 7y + 3z + 6=0


5x-7y-3z-6=0


For equation of plane Ax + By + Cz=D and point (x1,y1,z1), distance of a


point from a plane can be calculated as





Question 15.

Find the equation of the plane passing through the points A(1, 1, 2) and B(2, -2, 2) and perpendicular to the plane 6x – 2y + 2z = 9.


Answer:

Plane passes through (1,1,2) and (2,-2,2),

A(x-1) + B(y-1) + C(z-2)=0 (1)


A(x-2) + B(y + 2) + C(z-2)=0 (2)


Subtracting (1) from (2),


A(x-2-x + 1) + B(y + 2-y-1)=0


A-3B=0 (3)


Now plane is perpendicular to 6x-2y + 2z=9


6A-2B + 2C=0 (4)


Using (3) in (4)


18A-2B + 2C=0


16B + 2C=0


C=-8B


Putting values in equation (1)


3B(x-1) + B(y + 2)-8B(z-2)=0


B(3x-3 + y + 2-8z + 16)=0


3x + y-8z + 15=0



Question 16.

Find the equation of the plane passing through the points A(-1, 1, 1) and B(1, -1, 1) and perpendicular to the plane x + 2y + 2z = 5.


Answer:

Plane passes through (-1,1,1) and (1,-1,1),

A(x + 1) + B(y-1) + C(z-1)=0 (1)


A(x-1) + B(y + 1) + C(z-1)=0 (2)


Subtracting (1) from (2),


A(x-1-x-1) + B(y + 1-y + 1)=0


-2A + 2B=0


A=B (3)


Now plane is perpendicular to x + 2y + 2z=5


A + 2B + 2C=0 (4)


Using (3) in (4)


B + 2B + 2C=0


3B + 2C=0



Putting values in equation (1)



B(2(x + 1) + 2(y-1)-3(z-1)=0


2x + 2y-3z + 2-2-3=0


2x + 2y-3z-3=0



Question 17.

Find the equation of the plane through the points A( 3, 4, 2) and B(7, 0, 6) and perpendicular to the plane 2x – 5y = 15.

HINT: The given plane is 2x – 5y + 0z = 15


Answer:

Plane passes through (3,4,2) and (7,0,6),

A(x-3) + B(y-4) + C(z-2)=0 (1)


A(x-7) + B(y-0) + C(z-6)=0 (2)


Subtracting (1) from (2),


A(x-7-x + 3) + B(y-y + 4) + C(z-6-z + 2)=0


-4A + 4B-4C=0


A-B + C=0


B=A + C (3)


Now plane is perpendicular to 2x-5y=15


2A-5B=0 (4)


Using (3) in (4)


2A-5(A + C)=0


2A-5A-5C=0


-3A-5C=0




Putting values in equation (1)



A(5(x-3) + 2(y-4)-3(z-2)=0


5x + 2y-3z-15-8 + 6=0


5x + 2y-3z-17=0


So, required equation of plane is 5x + 2y-3z-17=0.



Question 18.

Find the equation of the plane through the points A(2, 1, -1) and B(-1, 3, 4) and perpendicular to the plane x – 2y + 4z = 10. Also, show that the plane thus obtained contains the line


Answer:

Plane passes through (2,1,-1) and (-1,3,4),

A(x-2) + B(y-1) + C(z + 1)=0 (1)


A(x + 1) + B(y-3) + C(z-4)=0 (2)


Subtracting (1) from (2),


A(x + 1-x + 2) + B(y-3-y + 1) + C(z-4-z-1)=0


3A-2B-5C=0 (3)


Now plane is perpendicular to x-2y + 4z=10


A-2B + 4C=0 (4)


Using (3) in (4)


2A-9C=0





Putting values in equation (1)



A(18(x-2) + 17(y-1) + 4(z + 1)=0


18x + 17y + 4z-36-17 + 4=0


18x + 17y + 4z-49=0


So, the required equation of plane is 18x + 17y + 4z-49=0


LHS=18(-1) + 17.3 + 4.4-49


=-18 + 51 + 16-49


=-2 + 2=0=RHS


In vector form normal of plane



LHS=18.3 + 17(-2) + 4.(-5)=54-34-20=0=RHS


Hence line is contained in plane.




Exercise 28g
Question 1.

Find the angle between the line and the plane


Answer:

Given - and


To find – The angle between the line and the plane


Direction ratios of the line = (1, - 1, 1)


Direction ratios of the normal of the plane = (2, - 1, 1)


Formula to be used – If (a, b, c) be the direction ratios of a line and (a’, b’, c’) be the direction ratios of the normal to the plane, then, the angle between the two is given by


The angle between the line and the plane







Question 2.

Find the angle between the line and the plane


Answer:

Given - and


To find – The angle between the line and the plane


Direction ratios of the line = (3, - 1, 2)


Direction ratios of the normal of the plane = (1, 1, 1)


Formula to be used – If (a, b, c) be the direction ratios of a line and (a’, b’, c’) be the direction ratios of the normal to the plane, then, the angle between the two is given by


The angle between the line and the plane






Question 3.

Find the angle between the line and the plane


Answer:

Given - and


To find – The angle between the line and the plane


Direction ratios of the line = (0, 1, 1)


Direction ratios of the normal of the plane = (2, - 1, 2)


Formula to be used – If (a, b, c) be the direction ratios of a line and (a’, b’, c’) be the direction ratios of the normal to the plane, then, the angle between the two is given by


The angle between the line and the plane






Question 4.

Find the angle between the line and the plane 3x + 4y + z + 5 = 0.


Answer:

Given - and


To find – The angle between the line and the plane


Direction ratios of the line = (3, - 1, 2)


Direction ratios of the normal of the plane = (3, 4, 1)


Formula to be used – If (a, b, c) be the direction ratios of a line and (a’, b’, c’) be the direction ratios of the normal to the plane, then, the angle between the two is given by


The angle between the line and the plane







Question 5.

Find the angle between the line and the plane 10x + 2y – 11z = 3.


Answer:

Given - and


To find – The angle between the line and the plane


Direction ratios of the line = (2, 3, 6)


Direction ratios of the normal of the plane = (10, 2, - 11)


Formula to be used – If (a, b, c) be the direction ratios of a line and (a’, b’, c’) be the direction ratios of the normal to the plane, then, the angle between the two is given by


The angle between the line and the plane







Question 6.

Find the angle between the line joining the points A(3, - 4, - 2) and B(12, 2, 0) and the plane 3x – y + z = 1.


Answer:

Given - A = (3, - 4, - 2) , B = (12, 2, 0) and 3x - y + z = 1


To find – The angle between the line joining the points A and B and the plane


Tip – If P = (a, b, c) and Q = (a’, b’, c’), then the direction ratios of the line PQ is given by ((a’ - a), (b’ - b), (c’ - c))


The direction ratios of the line AB can be given by


((12 - 3), (2 + 4), (0 + 2))


= (9, 6, 2)


Direction ratios of the normal of the plane = (3, - 1, 1)


Formula to be used – If (a, b, c) be the direction ratios of a line and (a’, b’, c’) be the direction ratios of the normal to the plane, then, the angle between the two is given by


The angle between the line and the plane






Question 7.

If the plane 2x – 3y – 6z = 13 makes an angle sin - 1 (λ) with the x - axis, then find the value of λ.


Answer:

Given - and


To find – The angle between the line and the plane


Direction ratios of the line = (1, 0, 0)


Direction ratios of the normal of the plane = (2, - 3, - 6)


Formula to be used – If (a, b, c) be the direction ratios of a line and (a’, b’, c’) be the direction ratios of the normal to the plane, then, the angle between the two is given by


The angle between the line and the plane





Question 8.

Show that the line is parallel to the plane Also, find the distance between them.


Answer:

Given - and


To prove – The line and the plane are parallel &


To find – The distance between them


Direction ratios of the line = (1, 3, 4)


Direction ratios of the normal of the plane = (1, 1, - 1)


Formula to be used – If (a, b, c) be the direction ratios of a line and (a’, b’, c’) be the direction ratios of the normal to the plane, then, the angle between the two is given by


The angle between the line and the plane






Hence, the line and the plane are parallel.


Now, the equation of the plane may be written as x + y - z = 7.


Tip – If ax + by + c + d = 0 be a plane and be a line vector, then the distance between them is given by


The distance between the plane and the line




units



Question 9.

Find the value of m for which the line is parallel to the plane


Answer:

Given - and and they are parallel


To find – The value of m


Direction ratios of the line = (2, - m, - 3)


Direction ratios of the normal of the plane = (m, 3, 1)


Formula to be used – If (a, b, c) be the direction ratios of a line and (a’, b’, c’) be the direction ratios of the normal to the plane, then, the angle between the two is given by







Question 10.

Find the vector equation of a line passing through the origin and perpendicular to the plane


Answer:

Given -


To find – The vector equation of the line passing through the origin and perpendicular to the given plane


Tip – The equation of a plane can be given by where is the normal of the plane


A line parallel to the given plane will be in the direction of the normal and will have the direction ratios same as that of the normal.


Formula to be used – If a line passes through the point (a, b, c) and has the direction ratios as (a’, b’, c’), then its vector equation is given by where λ is any scalar constant


The required equation will be


for some scalar λ



Question 11.

Find the vector equation of the line passing through the point with position vector and perpendicular to the plane


Answer:

Given - and the vector has position vector


To find – The vector equation of the line passing through (1, - 2, 5) and perpendicular to the given plane


Tip – The equation of a plane can be given by where is the normal of the plane


A line parallel to the given plane will be in the direction of the normal and will have the direction ratios same as that of the normal.


Formula to be used – If a line passes through the point (a, b, c) and has the direction ratios as (a’, b’, c’), then its vector equation is given by where λ is any scalar constant


The required equation will be for some scalar λ



Question 12.

Show that the equation ax + by + d = 0 represents a plane parallel to the z - axis. Hence, find the equation of a plane which is parallel to the z - axis and passes through the points A(2, - 3, 1) and B(- 4, 7, 6).


Answer:

Given – The equation of the plane is given by ax + by + d = 0


To prove – The plane is parallel to z - axis


Tip – If ax + by + cz + d is the equation of the plane then its angle with the z - axis will be given by


Considering the equation, the direction ratios of its normal is given by (a, b, 0)


The angle the plane makes with the z - axis = sin - 1[0/√(a2 + b2)] = 0


Hence, the plane is parallel to the z - axis


To find – Equation of the plane parallel to z - axis and passing through points A = (2, - 3, 1) and B = (- 4, 7, 6)


The given equation ax + by + d = 0 passes through (2, - 3, 1) & (- 4, 7, 6)


………(i)


……(ii)


Solving (i) and (ii),


[α → arbitrary constant]




Substituting the values of a and b in eqn (i), we get,


- 2Х10α + 3Х6α + d = 0 i.e. d = - 2α


Putting the value of a, b and d in the equation ax + by + d = 0,


(- 10α)x + (- 6α)y + (- 2α) = 0


i.e. 5x + 3y + 1 = 0


Question 13.

Find the equation of the plane passing through the points (1, 2, 3) and (0, - 1, 0) and parallel to the line


Answer:

Given – A plane passes through points (1, 2, 3) and (0, - 1, 0) and is parallel to the line


To find – Equation of the plane


Tip – If a plane passes through points (a’, b’, c’), then its equation may be given as a(x - a’) + b(y - b’) + c(z - c’) = 0


Taking points (1, 2, 3):


a(x - 1) + b(y - 2) + c(z - 3) = 0…………(i)


The plane passes through (0, - 1, 0):


a(0 - 1) + b(- 1 - 2) + c(0 - 3) = 0


i.e. a + 3b + 3c = 0………………(ii)


The plane is parallel to the line


Tip – The normal of the plane will be normal to the given line since both the line and plane are parallel.


Direction ratios of the line is (2, 3, - 3)


Direction ratios of the normal of the plane is (a, b, c)


So, 2a + 3b - 3c = 0………………………(iii)


Solving equations (ii) and (iii),


[α → arbitrary constant]





Putting these values in equation (i) we get,







Question 14.

Find the equation of a plane passing through the point (2, - 1, 5), perpendicular to the plane x + 2y - 3z = 7 and parallel to the line


Answer:

Given – A plane passes through (2, - 1, 5), perpendicular to the plane x + 2y - 3z = 7 and parallel to the line


To find – The equation of the plane


Let the equation of the required plane be ax + by + cz + d = 0……(a)


The plane passes through (2, - 1, 5)


So, 2a - b + 5c + d = 0…………………(i)


The direction ratios of the normal of the plane is given by (a, b, c)


Now, this plane is perpendicular to the plane x + 2y - 3z = 7 having direction ratios (1, 2, - 3)


So, a + 2b - 3c = 0……………(ii)


This plane is also parallel to the line having direction ratios (3, - 1, 1)


So, the direction of the normal of the required plane is also at right angles to the given line.


So, 3a - b + c = 0………………(iii)


Solving equations (ii) and (iii),


[α → arbitrary constant]





Putting these values in equation (i) we get,


2Х(- α) - (- 10α) + 5(- 7α) + d = 0 i.e. d = 27α


Substituting all the values of a, b, c and d in equation (a) we get,




Question 15.

Find the equation of the plane passing through the intersection of the planes
5x - y + z = 10 and x + y - z = 4 and parallel to the line with direction ratios
2, 1, 1. Find also the perpendicular distance of (1, 1, 1) from this plane.


Answer:

Given – A plane passes through the intersection of 5x - y + z = 10 and x + y - z = 4 and parallel to the line with direction ratios (2, 1, 1)


To find – Equation of the plane


Tip – If ax + by + cz + d = 0 and a’x + b’y + c’z + d’ = 0 be two planes, then the equation of the plane passing through their intersection will be given by


(ax + by + cz + d) + λ(a’x + b’y + c’z + d’) = 0, where λ is any scalar constant


So, the equation of the plane maybe written as


(5x - y + z - 10) + λ(x + y - z - 4) = 0


⇒ (5 + λ)x + (- 1 + λ)y + (1 - λ)z + (- 10 - 4λ) = 0


This is plane parallel to a line with direction ratios (2, 1, 1)


So, the normal of this line with direction ratios ((5 + λ), (- 1 + λ), (1 - λ)) will be perpendicular to the given line.


Hence,


2(5 + λ) + (- 1 + λ) + (1 - λ) = 0


⇒ λ = - 5


The equation of the plane will be


(5 + (- 5))x + (- 1 + (- 5))y + (1 - Χ(- 5))z + (- 10 - 4Χ(- 5)) = 0


⇒ - 6y + 6z + 10 = 0


3y - 3z = 5


To find – Perpendicular distance of point (1, 1, 1) from the plane


Formula to be used - If ax + by + c + d = 0 be a plane and (a’, b’, c’) be the point, then the distance between them is given by


The distance between the plane and the line




units




Exercise 28h
Question 1.

Find the vector and Cartesian equations of the plane passing through the origin and parallel to the vectors and


Answer:

Given - & are two lines to which a plane is parallel and it passes through the origin.


To find – The equation of the plane


Tip – A plane parallel to two vectors will have its normal in a direction perpendicular to both the vectors, which can be evaluated by taking their cross product






The plane passes through origin (0, 0, 0).


Formula to be used – If a line passes through the point (a, b, c) and has the direction ratios as (a’, b’, c’), then its vector equation is given by where λ is any scalar constant


The required plane will be




The vector equation :


The Cartesian equation : x + 2y + 3z = 0



Question 2.

Find the vector equation of a plane passing through the point (1, 2, 3) and parallel to the lines whose direction ratios are 1, - 1, - 2, and - 1, 0, 2.


Answer:

Given – The lines have direction ratios of (1, - 1, - 2) and (- 1, 0, 2). The plane parallel to these lines passes through (1, 2, 3)


To find – The vector equation of the plane


Tip – A plane parallel to two vectors will have its normal in a direction perpendicular to both the vectors, which can be evaluated by taking their cross product


& , where the two vectors represent the directions






The equation of the plane maybe represented as - 2x - z + d = 0


Now, this plane passes through the point (1, 2, 3)


Hence,


(- 2) × 1 - 3 + d = 0


⇒ d = 5


The Cartesian equation of the plane : - 2x - z + 5 = 0 i.e. 2x + z = 5


The vector equation :



Question 3.

Find the vector and Cartesian equations of the plane passing through the point
(3, - 1, 2) and parallel to the lines and


Answer:

Given - & . A plane is parallel to both these lines and passes through (3, - 1, 2).


To find – The equation of the plane


Tip – A plane parallel to two vectors will have its normal in a direction perpendicular to both the vectors, which can be evaluated by taking their cross product


& , where the two vectors represent the directions






The equation of the plane maybe represented as 4x + 5y - 17z + d = 0


Now, this plane passes through the point (3, - 1, 2)


Hence,


4 × 3 + 5 × (- 1) – 17 × 2 + d = 0


⇒ d = 27


The Cartesian equation of the plane : 4x + 5y - 17z + 27 = 0


The vector equation :



Question 4.

Find the Cartesian and vector equations of a plane passing through the point (1, 2, - 4) and parallel to the lines and


Answer:

Given - & . A plane is parallel to both these lines and passes through (1, 2, - 4).


To find – The equation of the plane


Tip – A plane parallel to two vectors will have its normal in a direction perpendicular to both the vectors, which can be evaluated by taking their cross product


The direction ratios of the given lines are (2, 3, 6) and (1, 1, - 1)


&






The equation of the plane maybe represented as - 9x + 8y - z + d = 0


Now, this plane passes through the point (1, 2, - 4)


Hence,


(- 9) × 1 + 8 × 2 - (- 4) + d = 0


⇒ d = - 11


The Cartesian equation of the plane : - 9x + 8y - z - 11 = 0 i.e. 9x - 8y + z + 11 = 0


The vector equation :



Question 5.

Find the vector equation of the plane passing through the point and parallel to the vectors and


Answer:

Given - & are two lines to which a plane is parallel and it passes through the point


To find – The equation of the plane


Tip – A plane parallel to two vectors will have its normal in a direction perpendicular to both the vectors, which can be evaluated by taking their cross product






The equation of the plane maybe represented as 5x + 2y - 3z + d = 0


Now, this plane passes through the point (3, 4, 2)


Hence,


5 × 3 + 2 × 4 – 3 × 2 + d = 0


⇒ d = - 17


The Cartesian equation of the plane : 5x + 2y - 3z - 17 = 0 i.e. 5x + 2y - 3z = 17


The vector equation :




Exercise 28i
Question 1.

Show that the lines and are coplanar.

Also find the equation of the plane containing these lines.


Answer:

Given : Equations of lines -




To Prove : are coplanar.


To Find : Equation of plane.


Formulae :


1) Cross Product :


If are two vectors




then,



2) Dot Product :


If are two vectors




then,



3) Coplanarity of two lines :


If two lines are coplanar then



4) Equation of plane :


If two lines are coplanar then equation of the plane containing them is



Where,



Answer :


Given equations of lines are




Let,


Where,






Now,





Therefore,



= 0 + 4 + 3


= 7


……… eq(1)


And



= - 2 + 12 – 3


= 7


……… eq(2)


From eq(1) and eq(2)



Hence lines are coplanar.


Equation of plane containing lines is



Now,



From eq(1)



Therefore, equation of required plane is







Question 2.

Find the vector and Cartesian forms of the equations of the plane containing the two lines and ..


Answer:

Given : Equations of lines -




To Find : Equation of plane.


Formulae :


1) Cross Product :


If are two vectors




then,



2) Dot Product :


If are two vectors




then,



3) Equation of plane :


If two lines are coplanar then equation of the plane containing them is



Where,


Given equations of lines are




Let,


Where,






Now,





Therefore,



= 6 – 56 – 48


= - 98


………eq(1)


Equation of plane containing lines is



Now,



From eq(1)



Therefore, equation of required plane is




This vector equation of plane.


As



= 6x – 28y + 12z


Therefore, equation of plane is


6x – 28y + 12z = -98


6x – 28y + 12z + 98 = 0


This Cartesian equation of plane.



Question 3.

Find the vector and Cartesian equations of a plane containing the two lines and Also show that the lines lies in the plane.


Answer:

Given : Equations of lines -




To Prove : are coplanar.


To Find : Equation of plane.


Formulae :


1) Cross Product :


If are two vectors




then,



2) Dot Product :


If are two vectors




then,



3) Coplanarity of two lines :


If two lines are coplanar then



4) Equation of plane :


If two lines are coplanar then equation of the plane containing them is



Where,



Answer :


Given equations of lines are




Let,


Where,






Now,





Therefore,



= 40 + 10 + 24


= 74


……… eq(1)


And



= 60 + 30 – 16


= 74


……… eq(2)


From eq(1) and eq(2)



Hence lines are coplanar.


Equation of plane containing lines is



Now,



From eq(1)


4


Therefore, equation of required plane is





This vector equation of plane.


As



= 20x + 10y – 8z


Therefore, equation of plane is


20x + 10y – 8z = 74


20x + 10y – 8z – 74 = 0


10x + 5y – 4z – 37 = 0


This Cartesian equation of plane.



Question 4.

Prove that the lines and are coplanar. Also find the equation of the plane containing these lines.


Answer:

Given : Equations of lines –


Line 1 :


Line 2 :


To Prove : Line 1 & line 2 are coplanar.


To Find : Equation of plane.


Formulae :


1) Coplanarity of two lines :


If two lines are given by,


and


, then these lines are coplanar, if



2) Equation of plane :


The equation of plane containing two coplanar lines


& is given by,



Answer :


Given lines –


Line 1 :


Line 2 :


Here, x1 = 0 , y1 = 2 , z1 = -3 , a1 = 1 , b1 = 2 , c1 = 3


x2 = 2 , y2 = 6 , z2 = 3 , a2 = 2 , b2 = 3 , c2 = 4


Now,






= - 2 + 8 – 6


= 0



Hence, given two lines are coplanar.


Equation of plane passing through line1 and line 2 is given by,






-x + 2y – 4 – z – 3 = 0


- x + 2y – z – 7 = 0


x – 2y + z + 7 = 0


Therefore, equation of plane is


x – 2y + z + 7 = 0



Question 5.

Prove that the lines and are coplanar. Also find the equation of the plane containing these lines.


Answer:

Given : Equations of lines –


Line 1 :


Line 2 :


To Prove : Line 1 & line 2 are coplanar.


To Find : Equation of plane.


Formulae :


1) Coplanarity of two lines :


If two lines are given by,


and


, then these lines are coplanar, if



2) Equation of plane :


The equation of plane containing two coplanar lines


& is given by,



Answer :


Given lines –


Line 1 :


Line 2 :


Here, x1 = 2 , y1 = 4 , z1 = 6 , a1 = 1 , b1 = 4 , c1 = 7


x2 = -1 , y2 = -3 , z2 = -5 , a2 = 3 , b2 = 5 , c2 = 7


Now,






= 21 - 98 + 77


= 0



Hence, given two lines are coplanar.


Equation of plane passing through line 1 and line 2 is given by,






-7x + 14 + 14y - 56 – 7z + 42 = 0


- 7x + 14y – 7z = 0


x – 2y + z = 0


Therefore, equation of plane is


x – 2y + z = 0



Question 6.

Show that the lines and are coplanar. Find the equation of the plane containing these lines.


Answer:

Given : Equations of lines –


Line 1 : or


Line 2 : or


To Prove : Line 1 & line 2 are coplanar.


To Find : Equation of plane.


Formulae :


1) Coplanarity of two lines :


If two lines are given by,


and


, then these lines are coplanar, if



2) Equation of plane :


The equation of plane containing two coplanar lines


& is given by,



Answer :


Given lines –


Line 1 :


Line 2 :


Here, x1 = 5 , y1 = 7 , z1 = -3 , a1 = 4 , b1 = 4 , c1 = -5


x2 = 8 , y2 = 4 , z2 = 5 , a2 = 7 , b2 = 1 , c2 = 3


Now,






= 51 + 141 – 192


= 0



Hence, given two lines are coplanar.


Equation of plane passing through line1 and line 2 is given by,






17x – 85 – 47y + 329 – 24z – 72 = 0


17x - 47y – 24z + 172 = 0


Therefore, equation of plane is


17x - 47y – 24z + 172 = 0



Question 7.

Show that the lines and are coplanar. Find the equation of the plane containing these lines.


Answer:

Given : Equations of lines –


Line 1 :


Line 2 :


To Prove : Line 1 & line 2 are coplanar.


To Find : Equation of plane.


Formulae :


1) Coplanarity of two lines :


If two lines are given by,


and


, then these lines are coplanar, if



2) Equation of plane :


The equation of plane containing two coplanar lines


& is given by,



Answer :


Given lines –


Line 1 :


Line 2 :


Here, x1 = -1 , y1 = 3 , z1 = -2 , a1 = -3 , b1 = 2 , c1 = 1


x2 = 0 , y2 = 7 , z2 = -7 , a2 = 1 , b2 = -3 , c2 = 2


Now,






= 7 + 28 – 35


= 0



Hence, given two lines are coplanar.


Equation of plane passing through line1 and line 2 is given by,






7x + 7 + 7y – 21 + 7z + 14 = 0


7x + 7y + 7z = 0


x + y + z = 0


Therefore, equation of plane is



Question 8.

Show that the lines and are coplanar. Also find the equation of the plane containing these lines.


Answer:

Given : Equations of lines –


Line 1 :


Line 2 :


To Prove : Line 1 & line 2 are coplanar.


To Find : Equation of plane.


Formulae :


1) Coplanarity of two lines :


If two lines are given by,


and


, then these lines are coplanar, if



2) Equation of plane :


The equation of plane containing two coplanar lines


& is given by,



Answer :


Given lines –


Line 1 :


Line 2 :


Here, x1 = 1 , y1 = 3 , z1 = 0 , a1 = 2 , b1 = -1 , c1 = -1


x2 = 4 , y2 = 1 , z2 = 1 , a2 = 3 , b2 = -2 , c2 = -1


Now,






= - 2



Hence, given two lines are not coplanar.



Question 9.

Find the equation of the plane which contains two parallel lines given by and


Answer:

Given : Equations of lines –


Line 1 :


Line 2 :


To Find : Equation of plane.


Formulae :


Equation of plane :


The equation of plane containing two parallel lines


& is given by,



Answer :


Given lines –


Line 1 :


Line 2 :


Here, x1 = 3 , y1 = -2 , z1 = 0 , a= 1 , b = -4 , c = 5


x2 = 4 , y2 = 3 , z2 = 2


Therefore, equation of plane containing line 1 & line 2 is given by,







33x – 99 – 3y – 6 – 9z = 0


33x – 3y – 9z – 105 = 0


11x – y – 3z = 35


Therefore, equation of plane is